ATI Pharmacology Questions

Ace your homework & exams now with Quizwiz!

A nurse is planning to administer epoetin alfa to a client who has chronic kidney failure. Which of the following should the nurse plan to review prior to administering this medication?

blood pressure

A nurse is teaching a client who has osteoporosis and a new prescription for alendronate. Which of the following client statements indicates that the teaching was effective?

"I will sit upright for 30 minutes after taking the medication"

A nurse is assessing an infant during a routine checkup. The parent asks the nurse about the infant's immunization schedule. Which of the following responses should the nurse make?

"It is recommended that your infant receives 6 immunizations at 2 months of age"

A nurse is administering a client's first dose of sucralfate. Which of the following explanations should the nurse provide about the action of sucralfate?

"Sucralfate forms a gel-like substance that protects ulcers"

A nurse is teaching a client about a new prescription for extended release oxycodone for pain management. Which of the following statements should the nurse include in the teaching?

"Swallow this medication whole"

A nurse is teaching a client who is experiencing age related vaginal atrophy and has a prescription for estradiol cream. Which of the following statements should the nurse include in the teaching?

"This medication has fewer systemic effects than oral estrogen"

A nurse is providing teaching to a client who has a prescription for ranitidine to treat a gastric ulcer. Which of the following statements should the nurse include in the teaching?

"This medication is less effective for people who smoke"

A nurse is providing teaching for a client who has a new prescription for nitroglycerin administered through a transdermal patch. Which of the following client statements indicates an understanding of the teaching?

"I need to rotate the location of my patch every few days"

A nurse is teaching a client who has been taking an NSAID to treat RA. During the client's first month checkup, the provider prescribed methotrexate to be added to the medication regimen. Which of the following statements should the nurse include in the teaching about the purpose of this change in the client's medication?

"This medication was added to delay the disease progression."

A nurse is providing teaching to a client with a seizure disorder who has a new prescription for carbamazepine. Which of the following statements should the nurse include in the teaching?

"This medication will decrease the effectiveness of oral contraceptives"

A nurse is teaching about a new prescription for ciprofloxacin to a client who has a urinary tract infection. The nurse should identify which of the following statements as an indication that the client understands the teaching?

"I will report any signs of tendon pain or swelling" Ciprofloxacin, a fluoroquinolone, is associated with a risk of tendon rupture. This risk is increased in older adult clients, so the client should notify the provider at the onset of tendon pain or swelling.

A nurse is teaching a female client about vit A supplementation. Which of the following client statements indicates an understanding of the teaching?

"A deficiency of vit A can cause night blindness."

A nurse is teaching about levodopa with a family member of a client who has Parkinson's disease. Which of the following pieces of info should the nurse include?

"A full therapeutic response may take several months to happen"

A nurse has administered a medication to a client. Which of the following circumstances should the nurse identify as a medication error that resulted form a performance deficit by the nurse?

"An IM injection was given instead of a subq injection"

A nurse is providing teaching about food-drug interactions to a client who is prescribed sirolimus following a kidney transplant. Which of the following pieces of info should the nurse include in the teaching?

"Avoid eating grapefruit while taking sirolimus"

A nurse is caring for a client who is refusing to take their scheduled morning furosemide. Which of the following statements should the nurse make?

"By not taking your furosemide, you might retain fluid and develop swelling." The nurse should respect the client's right to refuse the medication and inform the client of the risks of not taking the medication, notify the provider, and document the refusal. Furosemide is a loop diuretic given to reduce edema.

A nurse is teaching a client who has primary adrenal insufficiency (addison's disease) and a prescription for hydrocortisone. Which of the following statements should the nurse include in the teaching about this medication?

"Carry a supply of pills and a a single-use injectable preparation with you at all times"

A nurse is administering ciprofloxacin and phenaxopyridine to a client who has a severe UTI. The client asks why both medications are needed. Which of the following responses should the nurse make?

"Ciprofloxacin hydrochloride treats the infection and phenazopyridine treats pain."

A nurse is providing discharge teaching for a client who has a new prescription for metoprolol. Which of the following instructions should the nurse include?

"Do not stop taking this medication abruptly, count your radial pulse daily, change positions slowly."

A nurse is planning discharge teaching for a client who has a prescription for furosemide. The nurse should plan to include which of the following statements in the teaching?

"Drink a glass of milk with each dose of medication." The client should take furosemide with food or milk to reduce gastric irritation.

A nurse is teaching a client who has a new prescription for EC aspirin as stroke prophylaxis. The client asks the nurse why the provider prescribed an EC medication. Which of the following responses should the nurse give?

"EC medication cause less gastric irritation"

A nurse is caring for a client who is pregnant and inquiring about alternative, non-pharmacological therapies for N/V of pregnancy. Which of the following options should the nurse recommend?

"Ginger is effective in the tx of N/V"

A nurse is caring for a client who takes warfarin to treat chronic afib and has early manifestations of Alzheimer's disease. The client's partner asks the nurse if the client would benefit form taking ginkgo biloba. Which of the following responses should the nurse make?

"Ginkgo biloba will likely interfere with the effectiveness of his other medicaitons"

A nurse is caring for a client with MS and neurogenic bladder who is receiving bethanechol. The nurse should identify that which of the following client statements indicates a therapeutic action of the medication?

"I am able to urinate more feely"

A nurse in a provider's office is assessing a client who has been taking feverfew. Which of the following statements by the client indicates a therapeutic effect of the supplement?

"I am having fewer migraine headaches since I started taking feverfew"

A nurse is providing discharge teaching to a client who had a kidney transplant and has a prescription for oral cyclosporine. Which of the following statements by the client indicates an understanding of the teaching?

"I am likely to develop higher blood pressure while taking this medication"

A nurse is providing discharge teaching about lithium toxicity to a client who has a new prescription for lithium. Which of the following statements by the client indicates an understanding of the teaching?

"I can develop lithium toxicity if I experience vomiting or diarrhea."

A nurse is providing teaching about sodium phosphate to a client who has a new prescription for sodium phosphate. The client is scheduled for a colonoscopy and is currently taking furosemide for HTN. Which of the following client statements should indicate to the nurse that the teaching has been effective?

"I can experience an imbalance in my electrolytes from this medication"

A nurse is teaching a client about the proper placement of a nitroglycerin patch. Which of the following statements by the client indicates an understanding of the teaching?

"I can place the patch on any area of my body without hair"

A nurse is assessing a client with RA who has been taking high doses of prescribed hydroxychloroquine. Which of the following client statements should indicate to the nurse that the client is experiencing an adverse effect of the medication?

"I have had a change in my vision recently"

A nurse is monitoring a client who received diphenoxylate-atropine. Which of the following statements by the client should indicate to the nurse that the medication has been effective?

"I have not had a BM today"

A nurse is caring for a client who as TB and is taking rifampin. Which of the following client statements should indicate to the nurse that the client is experiencing an adverse effect of the medication?

"I have noticed my urine is orange in color."

A nurse at an urgent care clinic is collecting a history from a female client who has a urinary tract infection. The nurse anticipates a prescription for ciprofloxacin. The nurse should identify that which of the following client statements indicates a contraindication for administering this medication? "I have tendonitis, so I haven't been able to exercise." "I take a stool softener for chronic constipation." "I take medicine for my thyroid." "I am allergic to sulfa."

"I have tendonitis, so I haven't been able to exercise." The nurse should identify tendonitis as a contraindication for taking ciprofloxacin due to the risk of tendon rupture.

A nurse is providing teaching to a client about a new prescription for captopril to treat HTN. Which of the following statements indicates an understanding the teaching?

"I might feel dizzy at times while taking this medication"

A nurse is providing teaching to a client with HTN and type 1 DM who has a new prescription for metoprolol. Which of the following statements by the client indicates an understanding of the teaching?

"I might have difficulty recognizing when my blood sugar is low"

A nurse is teaching a client who has a prescription for scopolamine patches for the tx of motion sickness. Which of the following client statements should indicate to the nurse that the teaching has been effective?

"I should apply this patch behind my ear"

A nurse is teaching a female client who has a new prescription for misoprostol to treat PUD. Which of the following client statements should indicate to the nurse that the teaching was effective?

"I should avoid becoming pregnant while taking this medication"

A nurse is teaching a client who has a new diagnosis of angina and has a prescription for isosorbide mononitrate 10 mg PO BID. Which of the following client statements indicates an understanding of the teaching?

"I should change positions slowly when getting out of bed"

A nurse is teaching a client who has a new prescription for disulfiram to treat alcohol use disorder. Which of the following statements by the client indicates an understanding of the teaching?

"I should check the labels of my skin care products, medications, and food for alcohol"

A nurse is teaching a client who has a new prescription for alosetron. Which of the following client statements indicates an understanding of the teaching?

"I should contact my provider immediately if I experience constipation"

A nurse is teaching a client who has a new ezetimibe prescription for hyperlipidemia. Which of the following client statements should indicate to the nurse that the teaching was effective?

"I should let my doctor know if I have yellowing of my eyes"

A nurse is teaching about the adverse effects of baclofen with a client who has MS with spasms. Which of the following statements should the nurse identify as an indication that the client understands the teaching?

"I should not stop taking this medication suddenly."

A nurse is providing teaching to a client who has a UTI and new prescription for phenazopyridine and ciprofloxacin. Which of the following statements by the client indicates a need for further teaching?

"I should notify my provider immediately if my urine turns an orange color"

A nurse is teaching a client who will be taking dexamethasone daily for pain due to spinal edema. The nurse should identify which of the following client statements as an indication that the teaching has been effective?

"I should stay away from people who are ill"

A nurse is teaching a client who has systemic lupus erythematosus about a new prescription for oral glucocorticoid therapy. Which of the following client statements indicates an understanding of the teaching?

"I should take a calcium supplement while on this medication."

A nurse is providing teaching to a client who has depression and a new prescription for fluoxetine. Which of the following statements by the client indicates an understanding of the teaching?

"I should take acetaminophen instead of ibuprofen for my headaches while taking this medication" Fluoxetine suppresses platelet aggregation, which increases the risk of bleeding when used concurrently with NSAIDs and anticoagulants. Therefore, clients who are taking fluoxetine should take acetaminophen for headaches or pain, since acetaminophen does not suppress platelet aggregation.

A nurse is teaching a client who has a new prescription for amitriptyline to treat depression. Which of the following client statements indicates an understanding of the teaching?

"I should take this medication before bedtime"

A nurse is providing teaching to a client with chronic bronchitis about administering acetylcysteine using a hand held nebulizer. Which of the following client statements indicates an understanding of the teaching?

"I should try to cough productively just before I begin the treatment"

A nurse is teaching a client who has a new prescription for sucralfate for a duodenal ulcer. Which of the following client statements indicates an understanding of the teaching?

"I should wait at least 30 minutes before taking this medication after I take an antacid"

A nurse is teaching a client who is starting to take diltiazem. Which of the following statements should the nurse identify as an indication that the client understands the teaching?

"I will check my HR before I take the medication" Diltiazem, a calcium channel blocker, has cardio-suppressant effects at the SA and AV nodes, which can lead to bradycardia. The client should check their heart rate before taking the medication and notify the provider if it falls below the expected reference range.

A nurse is providing discharge teaching to a client who has HF and a prescription for digoxin 0.125 mg PO daily and furosemide 20 mg PO daily. Which of the following statements by the client indicates an understanding of the teaching?

"I will eat fruits and vegetables that have a high potassium content every day"

A nurse is providing teaching to a parent of a child who has asthma and a new prescription for a cromolyn sodium MDI. Which of the following statements by the parent indicates the need for further teaching?

"I will give my child a dose as soon as wheezing starts"

A nurse is providing teaching to a client who has a new prescription for a fentanyl transdermal patch. Which of the following statements by the client indicates an understanding of the teaching?

"I will have to stop drinking grapefruit juice while using the patch"

A nurse is providing teaching for a client who has gout and a prescription for allopurinol. Which of the following statements by the client should indicate to the nurse that the teaching was effective?

"I will increase my fluids to at least 2 liters per day"

A nurse is providing discharge teaching to a client who has been hospitalized for major depressive disorder and has a prescription for amitriptyline. Which of the following statements by the client indicates an understanding of the teaching?

"I will move slowly when I stand up because amitriptyline can cause my blood pressure to decrease"

A nurse is teaching a client who is about to start taking propylthiouracil to treat hyperthyroidism. Which of the following statements should the nurse identify as an indication that the teaching has been effective?

"I will need laboratory tests to check my liver function."

A nurse is providing teaching to a client who has postmenopausal osteoporosis and a new prescription for intranasal calcitonin-salmon. Which of the following statements by the client indicates an understanding of the teaching?

"I will need to depress the side arms to activate the pump"

A nurse is teaching about self-administration of transdermal medication with a male client who has a new prescription for nitroglycerin. The nurse should identify that which of the following statements by the client indicates an understanding of the teaching?

"I will remove the patch after 14 hours." The client should remove the patch after 12 to 14 hr to prevent tolerance of the medication.

A nurse is teaching a client who is starting PCA following a procedure. Which of the following client statements indicates an understanding of the teaching?

"This method works by keeping my opioid levels steady"

A nurse is instructing a client on the application of nitroglycerin transdermal patches. Which of the following statements by the client indicates an understanding of the teaching? "I should apply a patch every 5 minutes if I develop chest pain." "I will take the patch off right after my evening meal." "I will leave the patch off at least 1 day each week." "I should discard the used patch by flushing it down the toilet."

"I will take the patch off right after my evening meal." Clients should remove the patch each evening for a medication free time of 12 to 14 hr before applying a new patch to avoid developing a tolerance to the medication's effects.

A nurse is teaching a client who has osteoporosis about a new prescription for risedronate. Which of the following client statements indicates an understanding of the teaching?

"I will take this medication with a full cup of water"

A nurse is teaching a client about cyclobenzaprine. Which of the following client statements should indicate to the nurse that the teaching is effective? "I will have increased saliva production." "I will continue taking the medication until the rash disappears." "I will taper off the medication before discontinuing it." "I will report any urinary incontinence."

"I will taper off medication before discontinuing" The client should taper off cyclobenzaprine before discontinuing it to prevent abstinence syndrome or rebound insomnia.

A nurse is providing teaching to a client who has HF and is taking spironolactone. Which of the following statements by the client indicates an understanding of the teaching?

"I will watch for increased breast tissue growth while taking this medication"

A nurse is teaching a client who has a new prescription for warfarin. Which of the following statement should the nurse identify as an indication that the client understands the instructions?

"I'll avoid contact sports like football."

A nurse is teaching a client about the adverse effects of ompeprazole. Which of the following client statements indicates an understanding of the teaching?

"If I experience severe diarrhea, I will call my doctor"

A nurse is teaching about adverse effects of ergotamine with a client who has migraine HA. Which of the following client statements should indicate an understanding of the teaching?

"If I overuse this medication, I might become addicted to it"

A nurse is teaching a client who has severe chronic gout and a new prescription for pegloticase. The client has been taking allopurinol for 1 month. Which of the following instructions should the nurse include about pegloticase?

"If you experience a flare-up, you can take an NSAID while receiving this medication"

A nurse is teaching a client who has type 2 DM about a prescription for insulin lispro. Which of the following statements should the nurse include in the teaching?

"Insulin lispro has an onset of about 15 minutes"

A nurse is preparing to administer warfarin to a client who has a new onset of afib. The client asks the nurse, "what should this medication do?" Which of the following responses should the nurse make?

"It can reduce your risk of having a stroke"

A nurse is teaching a client about the use of a dinoprostone vaginal insert pouch to stimulate labor. Which of the following statements should the nurse include in the teaching?

"Lie on your back for at least 2 hours without getting up"

A nurse is teaching a client about taking tetracycline PO. Which of the following statements should the nurse include in the teaching?

"Limit your consumption of dairy products while taking this medicine"

A nurse is teaching a client who has a prescription for chenodiol for the treatment of gallstones. Which of the following client statements indicates an understanding of the teaching?

"Liver function tests are required while taking this medication"

A nurse is providing teaching to a client who has chronic constipation and a new prescription for psyllium. Which of the following instructions should the nurse provide?

"Mix this medication with water and follow with an additional glass of liquid"

A nurse is caring for a client who has MS and is receiving interferon beta-1a. The nurse should identify that which of the following client statements indicates a potential adverse effect of the medication?

"My body aches all over"

A nurse is assessing a client who has been taking simvastatin to treat hyperlipidemia. Which of the following statements by the client indicates an adverse effect of the medication that should be reported to the provider immediately?

"My legs feel weak and achy"

A nurse is caring for a client with COPD who has been taking tiotropium. Which of the following client statements should indicate to the nurse that the client is experiencing and adverse effect of this medication?

"My mouth feels dry all the time"

A nurse is caring for a client who is receiving chlorpromazine to treat schizophrenia. Which of the following statements by the client should prompt the nurse to notify the provider immediately?

"My tongue keeps moving like a worm."

A nurse is teaching a client who has persistent CA pain about the adverse effects of opioids. Which of the following statements should the nurse include in the teaching?

"Physical dependence is not the same as addiction"

A nurse is providing teaching about benzos to a client who is discontinuing long term alprazolam use. Which of the following pieces of info should the nurse include in the teaching?

"Plan to taper the dose slowly over several months"

A nurse is educating a client with urethritis who has a new prescription for oral erythromycin. Which of the following statements should the nurse include in the teaching?

"Report persistent diarrhea to the provider"

A nurse is caring for t a client who has been taking isoniazid and rifampin for 3 weeks for the tx of active pulmonary TB. The client reports his urine in an orange color. Which of the following statements should the nurse make?

"Rifampin can turn body fluids orange."

A nurse is teaching a client who has asthma and a prescription for a fluticasone dry powder inhaler. Which of the following instructions should the nurse include in the teaching?

"Rinse your mouth after administering this medication"

A nurse is providing discharge teaching to a client who has a bacterial infection about adverse effects of imipenem to report to the provider. Which of the following pieces of info should the nurse include?

"Seizures can occur with this medication"

A nurse is providing teaching for a client who has received a liver transplant and has a prescription to transition from cyclosporine to tacrolimus. Which of the following instructions should the nurse include in the teaching?

"Stop taking the cyclosporine for 24 hours and then begin taking the tacrolimus"

A nurse is teaching a client who has dyspepsia about prescribed antacids. Which of the following statements should the nurse include in the teaching?

"Take antacids 1 hour apart from other meds"

A nurse is providing teaching to a client who has a new prescription for hydrochlorothiazide 50 mg PO daily to treat HTN. Which of the following instructions should the nurse include in the teaching?

"Take the medication in the morning"

A nurse is teaching a client who is postmenopausal and has a prescription for alendronate. Which of the following statements should the nurse include in the teaching?

"Take this medication on an empty stomach"

A nurse is teaching the guardian of an infant about the diptheria, tetanus, and pertussis (DTaP) vaccine. Which of the following pieces of info should the nurse include in the teaching?

"The first immunization for DTaP in the series is given at 2 months"

A nurse is teaching the guardian of a school aged child about growth hormone therapy. Which of the following statements should the nurse include in the teaching?

"The hormone injections are administered subq"

A nurse is teaching a client with chronic asthma who has a new prescription for cromolyn. Which of the following instructions should the nurse include in the teaching?

"The medication's therapeutic effects can take up to several weeks to develop."

A nurse is preparing to administer the first injection of the DTaP vaccine to an infant. Which of the following pieces of info should the nurse tell the guardian prior to administering the immunization?

"The vaccine will be injected into the infant's thigh"

A nurse is teaching a client who has DM about a new prescription for pioglitazone. Which of the following statements should the nurse include in the teaching?

"This medication can be taken when using insulin"

A nurse is providing teaching to a client who is scheduled to start taking hydrochlorothizide for HTN. The nurse instructs the client to eat foods that are rich in potassium. Which of the following statements by the client indicates an understanding of the teaching?

"This medication can cause a loss of potassium"

A nurse is teaching a client with a new diagnosis of PUD who has a prescription for bismuth subsalicylate. The client asks the nurse, "how will this medication help my ulcer?" Which of the following statements should the nurse make?

"This medication can decrease bacteria in the GI tract"

A nurse is providing discharge teaching to a client who has angina pectoris and a new prescription for verapamil. The client tells the nurse. "My brother takes verapamil for high blood pressure. Do you think the provider made a mistake?" Which of the following responses should the nurse make?

"Verapamil is used to treat both high blood pressure and angina"

A nurse is teaching a client who has a prescription for doxycycline for the tx of a h. pylori infection. Which of the following instructions should the nurse include in the teaching?

"Wear protective clothing while in the sun"

A nurse is teaching a client who has a prescription for a combination oral contraceptive that uses a 28 day cycle. Which of the following instructions should the nurse include in the teaching?

"You can miss up to 7 pills with little risk in getting pregnant as long as you have taken the pills regularly for the previous 3 weeks"

A nurse is teaching a client who has allergic rhinitis about a new prescription for brompheniramine. Which of the following pieces of info should the nurse include in the teaching?

"You might find that you develop a dry mouth."

A nurse is providing teaching to a client who has MS and a new prescription for baclofen PO. Which of the following pieces of info should the nurse include?

"You should change positions slowly while taking this medication."

A nurse is providing teaching to a client with a new diagnosis of HF who has a prescription for furosemide. Which of the following statements should the nurse include in the teaching?

"You should eat foods that are high in potassium while taking this medication."

A nurse is caring for a client with premenstrual disorder who has a prescription for fluoxetine. The client asks the nurse, "When should I notice the benefits of this medication?" Which of the following responses should the nurse make?

"You should expect decreased manifestations within a few days"

A nurse is teaching about the adverse effects of morphine with a client who has acute pain. Which of the following statement should the nurse include in the teaching?

"You should increase your fluid intake"

A nurse is providing teaching about antiretroviral medication therapy to a client who has a new diagnosis of AIDS. Which of the following statements should the nurse include in the teaching?

"You should take antiretroviral medications on a routine schedule"

A nurse is providing discharge teaching to a client who had a bleeding duodenal ulcer and has been prescribed omeprazole. Which of the following statements should the nurse include in the teaching?

"You should take this medication before breakfast every day"

A nurse is teaching about taking donepezil with a client who was recently diagnosed with early Alzheimer's disease. Which of the following instructions should the nurse include in the teaching?

"You should take this medication late in the evening"

A nurse is providing teaching to a client who has TB and a prescription for isoniazid. Which of the following instructions should the nurse include?

"You should take this medication on an empty stomach"

A nurse is caring for a client who is taking diphenhydramine for allergies. The client reports "I feel sleepy during the day" Which of the following responses should the nurse make?

"You should try antihistamines with non-sedative effects"

An 18 month old toddler who has Kawasaki disease. The child is receiving IV immune globulin. The guardian asks the nurse to administer the child's scheduled measles, mumps, rubella vaccine before discharge. Which of the following responses should the nurse provide?

"Your child will not be able to receive the MMR vaccine for at least 3 months after discharge"

A nurse is caring for a client who is due to receive general anesthesia. The client asks the nurse, "what is the difference between an analgesic and anesthesia?" Which of the following statements should the nurse make?

"anesthesia can cause LOC"

A nurse is administering an EC tab to a client and explaining the pharmaceutical preparation. Which of the following statements should the nurse make?

"you are less likely to have an upset stomach with this pill because of the coating on the tablet"

A nurse is collecting a medication history from a client who has a new prescription for lithium. The nurse should identify that the client should discontinue which of the following over the-counter medications? Aspirin Ibuprofen Ranitidine Bisacodyl

-Ibuprofen Most NSAIDs can significantly increase lithium levels. Therefore, the client should not take ibuprofen and lithium concurrently.

A nurse is preparing to administer cefaclor 750 mg PO in 3 divided doses. Cefaclor 500 mg/tab is available. How many tablets should the nurse administer with each dose?

0.5

A nurse is preparing to administer levothyroxine 12.5 mcg PO daily to a client who has hypothyroidism. 25 mcg/1 tab is available. How many tablets should the nurse administer per dose?

0.5

A nurse is preparing to administer prochlorperazine 2.5 mg IV. An injection of 5 mg/mL is available. How many mL should the nurse administer?

0.5

A nurse is preparing to administer heparin 12,000 units subq to a client every 8hr. 20,000 units/1mL is available. How many mL should the nurse administer per dose?

0.6

A nurse is preparing to administer 1 mg of enalapril via IV bolus to a client who is experiencing HTN. The amount available is enalapril 1.25 mg/mL. How many mL should the nurse plan to administer per dose?

0.8

A nurse is preparing to administer digoxin 0.2 mg via IV bolus to a client. The amount available is 0.25 mg/1 mL. How many mL should the nurse administer?

0.8

A nurse is preparing to administer heparin 8000 units subq every 8 hrs. 10000 units/1 mL is available. How many mL should the nurse administer per dose?

0.8

A nurse is caring for a client who received 0.9% sodium chloride 1 L over 4 hr instead of over 8 hr as prescribed. Which of the following information should the nurse enter as a complete documentation of the incident? IV fluid infused over 4 hr instead of the prescribed 8 hr. Client tolerated fluids well, provider notified. 0.9% sodium chloride 1 L IV infused over 4 hr. Vital signs stable, provider notified. 1 L of 0.9% sodium chloride completed at 0900. Client denies shortness of breath. IV fluid initiated at 0500. Lungs clear to auscultation.

0.9% sodium chloride 1 L IV infused over 4 hr. Vital signs stable, provider notified The nurse should document the type and amount of fluid, how long it took to infuse, provider notification, and the client's physical status.

A nurse is caring for a client who receives gastrostomy tube feedings and insulin. The client is scheduled to receive a tube feeding at 0700. At which of the following times should the nurse plan to administer insulin lispro subq?

0645

A nurse is administering insulin glulisine 10 units subq at 0730 to an adolescent client who has type 1 DM. The nurse should anticipate the onset of action of the insulin at which of the following times?

0745

A nurse is preparing to administer a scheduled antibiotic at 0800 to a client and discovers the antibiotic is not present in the client's medication drawer. The nurse should identify that administration of the medication can occur at which of the following time periods without requiring an incident report? 1000 0900 0830 1200

0830 The nurse should identify that an antibiotic can be administered 30 min before or after the scheduled time to maintain therapeutic blood levels without requiring an incident report.

A nurse is planning care for a client who is receiving gentamicin IM and has a new prescription to obtain gentamicin peak and trough levels. At which of the following times should the nurse plan to obtain a blood sample to evaluate the gentamicin peak?

1 hour after administrating the IM injection

A nurse is providing teaching for a client who has multiple sclerosis and a new prescription for methylprednisone. Which of the following instructions should the nurse include? (select all)

1) blood glucose levels will need to be monitored during therapy The nurse should instruct the client that their blood glucose levels will be monitored during therapy because corticosteroids, such as methylprednisolone, can raise blood glucose levels. 2) avoid contact with persons who have known infections The nurse should instruct the client to avoid contact with persons who have known infections because corticosteroids, such as methylprednisolone, suppress the immune response and mask manifestations of infection. 3) grapefruit juice can increase the blood levels of the medication The nurse should instruct the client that grapefruit juice increases the absorption of the medication, which can lead to toxicity and adrenal suppression.

A nurse is preparing to administer atropine 0.6 mg IM preoperatively to a client. The amount available is atroping 0.4 mg/1 mL. How many mL should the nurse plan to administer?

1.5

A nurse is preparing to administer meperidine 100 mg IM to a client who has a BMI of 23. Which of the following needle lengths should the nurse use to administer the medication?

1.5 inch

A nurse is preparing to administer desmopressin 0.3 mcg/kg in 0.9% sodium chloride 50 mL IV over 30 min to a client who weighs 154 lb. How many mcg of medication should the client receive?

21

A nurse is preparing to administer codeine 30 mg PO every 4 hr PRN to a client for pain. The amount availabe is 15 mg/5 mL. How mnay mL should the nurse plan to administer per dose?

10

A nurse is preparing to administer heparin 500 units/hr to a client who has a DVT. Heparin is available at 25,000 units in 500 mL of 5% dextrose in water. The nurse should set the IV pump to deliver how many mL/hr?

10

A nurse is preparing to administer dextrose 5% in water (D5W) 400 mL IV to infuse over 1 hr. The drop factor of the manual IV tubing is 15 gtt/mL. The nurse should set the manual IV infusion to deliver how many gtt/min? (Round the answer to the nearest whole number. Use a leading zero if it applies. Do not use a trailing zero.)

100 gtt/min

A nurse is preparing to administer fosphenytoin 550 mg via IV bolus to a client who is having a seizure. Fosphenytoin 50 mg/1 mL is available. How many mL should the nurse administer?

11

A nurse is preparing to administer D5W 1000 mL to infuse over 12 hr. The drop factor of the manual IV tubing is 10 gtt/mL. The nurse should set the manual IV infusion to deliver how many gtt/min?

14

A nurse is preparing to administer azithromycin 150 mg liquid suspension PO every 12 hours to a client. The amount available is 50 mg/5 mL. How many mL should the nurse administer per dose?

15

A nurse is preparing to administer heparin 900 units/hr via IV infusion. The amount available is 25000 units in 500 mL 5% dextrose in water. The nurse should set the IV pump to deliver how many mL/hr?

18

A nurse is preparing to administer 0.9 % sodium chloride (NaCl) 1,500 mL to infuse over 8 hr to a client who is postoperative. The nurse should set the IV pump to deliver how many mL/hr?

188 mL/hr

A nurse is preparing to administer D5W 1000 mL for infusion over 10 hr. The drop factor of the manual IV tubing is 15 gtt/mL. The nurse should set the manual IV infusion to deliver how many gtt/min?

25

A nurse is preparing to administer benztropine 8 mg PO daily in 2 divided doses to a client who has Parkinson's disease. The amount available is benztropine 2 mg tablets. How many tablets should the nurse administer with each dose?

2

A nurse is preparing to administer metoclopramide 10 mg IM to a client who is postop and nauseated. The amount available is 5 mg/1 mL. How many mL should the nurse administer?

2

A nurse is preparing to administer cefepime 1g in D5W 50 mL over 30 min to a client who has pneumonia. The drop factor of the manual IV tubing is 15 gtt/mL. The nurse should set the manual IV infusion to deliver how many gtt/min?

25

A nurse is caring for the parent of a newborn. The parent asks the nurse when their newborn should receive the first diphtheria, tetanus, and pertussis vaccine (DTaP). The nurse should instruct the parent that their newborn should receive the immunization at which of the following ages? At birth 2 months 6 months 15 months

2 months The CDC recommends that newborns receive the first dose of the five-dose series of the DTaP immunization at 2 months of age.

A nurse is preparing to administer LR 700 mL IV by infusion over 24 hr to a pediatric client. The drop factor of the manual IV tubing is 60 gtt/mL. The nurse should set the manual IV infusion to deliver how many gtt/min?

29

A nurse is preparing to administer verapamil 5.5 mg via IV bolus to a client who has HTN. The amount available is verapamil 2.5 mg/mL. How many mL should the nurse administer?

2.2

A nurse is preparing to administer chlorothiazide 20 mg/kg/day PO divided equally and administered twice daily for a toddler who weighs 28.6 lb. The amount available is an oral suspension of 250 mg/5 mL. How many mL should the nurse administer per dose?

2.6

A nurse is preparing to administer furosemide 4 mg/kg/day PO divided into 2 equal doses daily to a toddler who weighs 22 lb. How many mg should the nurse administer per dose?

20

A nurse is preparing to administer LR 1000 mL IV infused over 8 hr. The drop factor of the manual IV tubing is 10 gtt/mL. The nurse should set the manual IV infusion to deliver how many gtt/min?

21

A nurse is preparing to administer acetaminophen 1 g PO 3 times per day PRN to a client who has a fever. The amount available is 325 mg/1 tab. How many tab should the nurse administer per dose?

3 tab

A nurse is preparing to administer ciprofloxacin 15 mg/kg PO every 12 hr to a child who weighs 44 lb. How many mg should the nurse administer per dose? (Round the answer to the nearest whole number. Use a leading zero if it applies. Do not use a trailing zero.)

300 mg

A nurse is preparing to administer 0.9% sodium chloride 1,000 mL IV over 8 hr to a client. The drop factor of the manual IV tubing is 15 gtt/mL. The nurse should set the manual IV infusion to deliver how many gtt/min?

31 gtt/min Step 1: What is the unit of measurement the nurse should calculate? gtt/min Step 2: What is the volume the nurse should infuse? 1 L= 1,000 mL Step 3: What is the total infusion time? 8 hr Step 4: Should the nurse convert the units of measurement? Yes (min does not equal hr) 1 hr8 hr = 60 minX min X min = 480 min Step 5: Set up an equation and solve for X. 1,000 mL15 gttX gtt/min = × 480 min1 mL X gtt/min = 31.25 gtt/min Step 6: Round if necessary. 31.25 = 31 Step 7: Determine whether the amount to administer makes sense. If the prescription reads 0.9% sodium chloride 1,000 mL IV to infuse over 8 hr with a drop factor of 15 gtt/min, it makes sense to administer 31 gtt/min. The nurse should set the manual IV infusion to deliver 0.9% sodium chloride IV at 31 gtt/min.

A nurse is preparing to administer ampicillin 500 mg in 50 mL of dextrose 5% in water infused over 15 min. The drop factor of the manual IV tubing is 10 gtt/mL. The nurse should set the manual IV infusion to deliver how many gtt/min?

33

A nurse is preparing to administer magnesium hydroxide 1.5 oz PO to a client who has constipation. How many mL should the nurse administer?

45

A nurse is preparing to administer amoxicillin 250 mg liquid suspension PO every 8 hr to an older adult client. The amount available is amoxicillin 50 mg/mL. How many mL should the nurse administer per dose?

5

A nurse is preparing to administer 150 units/hr of regular insulin to a client. Regular insulin is available at 1500 units in 0.9% sodium chloride 500 mL. The nurse should set the IV pump to deliver how many mL/hr?

50

A nurse is preparing to administer dextrose 5% in 0.45% sodium chloride 400 mL IV to an older adult client over 8 hr. The nurse should set the IV pump to deliver how many mL/hr?

50

A nurse is preparing to administer an IV bolus of 1 L NS over 2 hr to a client who is dehydrated. The nurse should set the IV pump to deliver how many mL/hr?

500

A nurse is preparing to administer cefixime 4 mg/kg PO twice daily to a preschooler who weighs 31 lb. How many mg should the nurse administer with each dose?

56

A nurse is planning care for a client who is postop and scheduled to ambulate. At which of the following times should the nurse plan to administer PO morphine to the client for peak analgesic effect during the ambulation?

60-90 min prior to ambulation

A nurse is preparing to administer and enteral feeding through an NG tube at 250 mL over 4 hr. The nurse should set the pump to deliver how many mL/hr?

63

A nurse is preparing to administer ampicillin 50 mg/kg/day PO divided into 4 equal doses for a toddler who weighs 33 lb. 125 mg/5 mL oral solution is available. How many mL should the nurse administer per dose?

7.5

A charge nurse is teaching a newly licensed nurse about the purpose of a client being prescribed a transdermal fentanyl patch. Which of the following clients should the charge nurse include in the teaching as a client who requires this medication?

A client who is opioid-tolerant

A nurse is teaching a group of unit nurses about medication reconciliation. Which of the following information should the nurse include in the teaching? The client's provider is required to complete medication reconciliation. Medication reconciliation at discharge is limited to the medication ordered at the time of discharge. A transition in care requires the nurse to conduct medication reconciliation. Medical reconciliation is limited to the name of the medications that the client is currently taking.

A transition in care requires the nurse to conduct medication reconciliation The nurse should conduct medication reconciliation anytime the client is undergoing a change in care such as admission, transfer from one unit to another, or discharge. A complete listing of all prescribed and over-the-counter medications should be reviewed.

A nurse is preparing to administer iron dextran IV to a client. Which of the following actions should the nurse plan to take?

Administer a small test dose before giving the full dose

A nurse administered an antibiotic 10 min ago to a client who is now reporting wheezing and swelling of the eyelids. Which of the following actions should the nurse perform first?

Administer epinephrine subq

A nurse is caring for a child who has epilepsy and is scheduled to receive a dose of phenytoin. The nurse notes the child's serum phenytoin level is 14 mcg/mL. Which of the following actions should the nurse take?

Administer the dose

A nurse is administering subq heparin to a client who is at risk for DVT. Which of the following actions should the nurse take?

Administer the medication into the client's abdomen

A nurse is assessing a client who has a new prescription for chlorpromazine to treat schizophrenia. The client has a mask-like facial expression and is experiencing involuntary movements and tremors. Which of the following medications should the nurse anticipate administering?

Amantadine

A nurse is assessing a client who is experiencing chest pain. Which of the following medications should the nurse expect to administer to suppress the aggregation of platelets?

ASA

A nurse is preparing to administer medications to a client who is NPO and is receiving enteral feedings through an NG tube. Which of the following prescriptions should the nurse clarify with the provider?

ASA EC 325 mg per NG tube daily

A nurse is teaching a client who has a prescription for a transdermal estradiol patch. In which of the following locations should the nurse instruct the client to apply the patch?

Abdomen

A nurse is reviewing the laboratory data for a client who is receiving clozapine for schizophrenia. The nurse should identify which of the following findings as a potential adverse effect of the medication?

Absolute neutrophil count 1200 mm^3

A nurse is evaluating how a client who is pregnant is responding to a medication. Which of the following physiological effects of pregnancy should the nurse take into consideration?

Accelerated excretion of fluids

A nurse is caring for a client who has PUD and reports a HA. Which of the following medications should the nurse plan to administer?

Acetaminiophen

A nurse is caring for a client who has acute acetaminophen toxicity. The nurse should anticipate administering which of the following medications? Vitamin K Acetylcysteine Benztropine Physostigmine

Acetylcysteine Acetylcysteine is a specific antidote for acetaminophen toxicity. It can prevent severe injury when given orally or by IV infusion within 8 to 10 hr.

A nurse is preparing a discharge teaching plan for a 6 yo client with asthma who has several prescription medications using an MDI. Which of the following interventions should the nurse include in the plan?

Add a spacer to each MDI

A nurse is preparing to administer heparin subcutaneously to a client. Which of the following actions should the nurse plan to take? Administer the medication outside the 5-cm (2-in) radius of the umbilicus. Aspirate for blood return before injecting. Rub vigorously after the injection to promote absorption. Place a pressure dressing on the injection site to prevent bleeding.

Administer the medication outside the 5-cm (2-in) radius of the umbilicus. The nurse should administer the heparin by subcutaneous injection to the abdomen in an area that is above the iliac crest and at least 5 cm (2 in) away from the umbilicus.

A nurse is preparing to administer a hydromorphone IV infusion to a client for pain. Which of the following actions should the nurse take?

Administer the medication over 4-5 minutes

A nurse is completing an incident report for a medication error. Which of the following information should the nurse include in the report?

Administered propranolol 80 mg PO at 1800 to the client who did not have a prescription for the medication. The incident report should clearly and thoroughly report the facts of the error.

A nurse is administering a medication parenterally to a client. Which of the following techniques should the nurse use to reduce fluctuations in plasma medication levels?

Administering a continuous infusion of the dose

A nurse is precepting a newly licensed nurse who is caring for four clients. The nurse should complete an incident report for which of the following actions by the newly licensed nurse?

Administers isosorbide mononitrate to a client who has BP 82/60 mm Hg Isosorbide mononitrate is a nitrate used for clients with angina. Taking isosorbide mononitrate leads to vasodilation, which can result in hypotension. The nurse should withhold the medication and notify the provider if the client's systolic blood pressure is below the expected reference range of 120/80.

A nurse is caring for a client who is receiving haloperidol. The nurse should identify which of the following findings as an adverse effect of the medication?

Akathisia An adverse effect associated with haloperidol is the development of extrapyramidal manifestations such as dystonia, pseudoparkinsonism, and akathisia.

A nurse is caring for a client who has asthma and a prescription for zileuton. Which of the following lab values should the nurse monitor while the client is taking this medication?

Alanine aminotransferase (ALT)

A nurse is providing teaching to the parents of a school-age child with asthma about medications for bronchospasm. Which of the following inhaled medications should the nurse instruct the parents to use to relieve an acute asthma attack?

Albuterol

A nurse is reviewing the medication history of a client who has mild intermittent asthma. The nurse should anticipate a prescription for which of the following inhalers for the client?

Albuterol sulfate

A nurse in a provider's office is assessing a client who has been taking amoxicillin for 10 days and reports diarrhea and cramping. The nurse should recognize that these manifestations occur secondary to which of the following adverse effects?

Alterations in the GI flora

A nurse on a medical unit is preparing to administer alendronate 40 mg PO for an older adult client who has Paget's disease of the bone. Which of the following actions should be the nurse's priority?

Ambulate the client to a chair prior to administering the medication

A nurse is teaching a group of nurses about the manifestations of progestin deficiency for clients who take a combination oral contraceptive. Which of the following findings should the nurse include in the teaching as an indication of progestin deficiency?

Amenorrhea

A nurse is caring for a client who has COPD and has been taking fluticasone via inhaler for many years. Which of the following findings should the nurse identify as an adverse effect of long term use of this medication?

Anorexia and weakness

A nurse is teaching a client about warfarin. The client asks if they can take aspirin while taking the warfarin. Which of the following responses should the nurse make? "It is safe to take an enteric-coated aspirin." "Aspirin will increase the risk of bleeding." "Acetaminophen may be substituted for aspirin." "The INR lab work must be monitored more frequently if aspirin is taken."

Aspirin will increase the chances of bleeding Aspirin inhibits platelet aggregation and can potentiate the action of the anticoagulant warfarin. Therefore, the client should avoid taking aspirin because it increases the risk for bleeding.

A nurse is caring for a client who has a new prescription for tamoxifen. The nurse should recognize that tamoxifen has which of the following therapeutic effects?

Anti-estrogenic

A nurse is caring for a client who has a dry nonproductive cough. Which of the following types of medication should the nurse recommend?

Antitussive

A nurse is preparing to administer nitroglycerin topical ointment to a client who has angina. Which of the following actions should the nurse take?

Apply the ointment using a dose-measuring appicator.

A nurse is teaching a client who is using topical lidocaine about preventing systemic toxicity. Which of the following pieces of info should the nurse include about the application of topical lidocaine?

Apply topical lidocaine to affected areas that are intact

A nurse is caring for a client who is receiving lidocaine for localized pain. The nurse should recognize that which of the following actions will help prevent systemic toxicity of this medication?

Applying the medication to intact skin

A nurse working in the emergency department is admitting a client who has a gastric ulcer and GI bleeding. Which of the following factors in the client's medical history should the nurse report to the provider?

Arthritis treated with ibuprofen every 8 hours as needed

A nurse is caring for a client who has a positive TB skin test and is beginning a prescription for isoniazid. Which of the following lab values should be monitored while the client is taking isoniazid?

Aspartate aminotransferase (AST)

A nurse is planning to administer diphenhydramine 50 mg via IV bolus to a client who is having an allergic reaction. The client has an IV infusion containing a medication that is incompatible with diphenhyramine in solution. Which of the following actions should the nurse take?

Aspirate to check for IV patency before administering the medication

A nurse is caring for a client and realized after administering the 0900 medications that she administered digoxin 0.25 mg PO to the client instead of the prescribed digoxin 0.125 mg PO. Which of the following actions should the nurse take first?

Assess the client's apical pulse

A nurse is caring for a client who has multiple medication allergies. During which of the following steps of the nursing process should the nurse identify the client's allergies?

Assessment

A nurse is preparing to administer timolol eye drops to a client who has primary open angle glaucoma. Prior to administering the medication, the nurse should recognize that which of the following conditions in the client's medical history is a contraindication to receiving this medication?

Asthma

A nurse is reviewing the medical record of a client who has hypertension. The nurse should identify which of the following findings as a contraindication for receiving propranolol?

Asthma Asthma is a contraindication for receiving propranolol. Propranolol is an adrenergic antagonist which blocks the beta2 receptors in the lungs, causing bronchoconstriction and leading to serious airway resistance and possibly respiratory arrest.

A nurse in a postpartum unit is caring for a client who plans to breastfeed her newborn exclusively. The client has a prescription for depot medroxyprogesterone acetate. At which of the following times should the nurse schedule the client to receive the first dose of the medication?

At 6 weeks postpartum

A nurse in an emergency department is caring for a client who has myasthenia gravis and is in a cholinergic crisis. Which of the following medications should the nurse plan to administer? Potassium iodide Glucagon Atropine Protamine

Atropine A cholinergic crisis is caused by an excess amount of cholinesterase inhibitor, such as neostigmine. The nurse should plan to administer atropine, an anticholinergic agent, to reverse cholinergic toxicity.

A nurse is providing teaching to a client who has HTN and a new prescription for oral clonidine. Which of the following instructions should the nurse include in the teaching?

Avoid driving until the client's reaction to the medication is known.

A nurse is caring for a client who has a prescription for a QT interval medication. Which of the following conditions should the nurse identify as an adverse effect of this medication?

Bradycardia

A nurse is caring for a client with psudomonas infection who has a new pescription for ticarcillin-clavulanate. Which of the following data should the nurse collect before administering this medication?

Baseline BUN and creatinine

A nurse is planning care for a client who is receiving mannitol via continuous IV infusion. The nurse should monitor the client for which of the following adverse effects? Weight loss Increased intraocular pressure Auditory hallucinations Bibasilar crackles

Bibasilar crackles Mannitol, an osmotic diuretic, can precipitate heart failure and pulmonary edema. Therefore, the nurse should recognize lung crackles as an indicator of a potential complication and stop the infusion.

A nurse is caring for a client who is taking an agonist medication. The nurse should expect which of the following actions from this type of medication?

Binds to receptors and mimics regulatory molecules

A nurse in a provider's office is assessing a client who reports taking a dietary supplement to reduce hot flashes related to menopause. Which of the following supplements should the nurse expect the client to report taking?

Black cohosh

A nurse is caring for a client who had a MI 2 hours ago and is receiving alteplase. Which of the following findings should the nurse identify as an adverse effect of receiving this medication?

Bleeding

A nurse is caring for a client who has a prescription for clopidogrel. The nurse should monitor the client for which of the following adverse effects?

Bleeding

A nurse is caring for a client who is recovering from deep-vein thrombosis and is to start taking warfarin. For which of the following findings should the nurse monitor as an adverse effect of warfarin?

Bleeding gums The nurse should monitor the client for bleeding gums, which is an adverse effect of warfarin, an anticoagulant.

A nurse is providing teaching to a client who has multiple sclerosis and a new prescription for methylprednisolone. Which of the following instructions should the nurse include? (Select all that apply.) Blood glucose levels will be monitored during therapy. Avoid contact with people who have known infections. Take the medication 1 hr before breakfast. Decrease dietary intake of foods containing potassium. Grapefruit juice can increase the effects of the medication.

Blood glucose levels will be monitored during therapy. Avoid contact with people who have known infections Grapefruit juice can increase the effects of the medication. Blood glucose levels will be monitored during therapy is correct. The nurse should monitor the client for hyperglycemia while providing methylprednisolone to the client. Glucocorticoids, such as methylprednisolone, increase serum glucose levels and can require management with insulin or antihyperglycemics. Avoid contact with people who have known infections is correct. The nurse should instruct the client to avoid exposure to infectious agents, such as contact with those who have active infections or illnesses. Glucocorticoids, such as methylprednisolone, depress the immune system, placing the client at an increased risk for developing an infection. Take the medication 1 hr before breakfast is incorrect. The nurse should instruct the client to take the medication with food or milk to decrease gastrointestinal upset. Decrease dietary intake of foods containing potassium is incorrect. The nurse should instruct the client to increase dietary intake of potassium-rich foods while taking this medication. Glucocorticoids, such as methylprednisolone, deplete potassium in the body, which manifests as hypokalemia. Grapefruit juice can increase the effects of the medication is correct. The nurse should instruct the client that grapefruit and grapefruit juice can increase the level of methylprednisolone in the body.

A nurse is caring for a client who is receiving heparin therapy via continuous IV infusion to treat a pulmonary embolism. Which of the following findings should the nurse identify as an adverse effect of the medication and report to the provider? Vomiting Blood in the urine Positive Chvostek's sign Ringing in the ears

Blood in the urine The nurse should report blood in the urine to the provider because this can be a manifestation of heparin toxicity. Other manifestations can include bruising, hematomas, hypotension, and tachycardia.

A nurse is caring for a client who is receiving IV famotidine. Which of the following adverse effects should the nurse report to the provider immediately?

Bloody stools

A nurse is caring for a client who is experiencing cycloplegia following the administration of atropine eye drops during an eye examination. Which of the following findings should the nurse expect as a result of cyloplegia?

Blurred vision

A nurse is assessing a client who has received atropine eye drops during an eye examination. Which of the following findings should the nurse expect as an adverse effect of the medication?

Blurred vision Blurred vision is an expected finding following the administration of atropine eye drops. This is due to the cycloplegic effects of the medication, which cause distant objects to appear blurry to the client.

A nurse is providing teaching to a client who is to begin taking oxybutynin for urinary incontinence. Which of the following adverse effects should the nurse include in the teaching? (Select all that apply.) Dry mouth Tinnitus Blurred vision Bradycardia Dry eyes

Blurred vision, Dry eyes, Dry Mouth Dry mouth is correct. Oxybutynin is an anticholinergic agent that can cause dry mouth. Tinnitus is incorrect. Oxybutynin can cause several sensory adverse effects including increased intraocular pressure. The nurse should instruct the client to report eye pain, seeing colored halos around lights, and a decreased ability to perceive light changes. However, tinnitus is not an adverse effect associated with oxybutynin administration. Blurred vision is correct. Oxybutynin is an anticholinergic agent that can cause blurred vision due to an increase in intraocular pressure. Bradycardia is incorrect. Oxybutynin can cause several cardiovascular adverse effects such as a prolongation of the QT interval, palpitations, hypertension, and tachycardia. Dry eyes is correct. Oxybutynin is an anticholinergic agent that can cause dry eyes and mydriasis, or pupil dilation.

A nurse is planning discharge teaching for a client who has major depressive disorder and a new prescription for phenelzine. Which of the following foods should the nurse include in the plan as safe for the client to consume while taking phenelzine?

Broiled beef steak

A nurse is caring for a client who has bronchitis and a prescription for a mucolytic agent. Which of the following findings should the nurse identify as an adverse effect of this type of medication?

Bronchospasm

A nurse is caring for a client who takes sulfasalazine twice daily for RA. Which of the following values should the nurse review prior to the administration of the medication?

CBC

A nurse is caring for a client who has a magnesium level of 3.1 mEq/L. The nurse should expect to administer which of the following medications?

Calcium gluconate The nurse should expect to administer IV calcium gluconate to the client and prepare to provide ventilatory support. This client is at risk for respiratory depression and cardiac dysrhythmias because a magnesium level of 3.1 mEq/L is above the expected reference range of 1.3 to 2.1 mEq/L.

A nurse is reviewing the medication list of a client who wants to begin taking oral contraceptives. The nurse should identify that which of the following client medications will interfere with the effectiveness of oral contraceptives? Carbamazepine Sumatriptan Atenolol Glipizide

Carbamazepine Carbamazepine causes an accelerated inactivation of oral contraceptives because of its action on hepatic medication-metabolizing enzymes.

A nurse is monitoring a client who is receiving phenytoin IV for the treatment of status epilepticus. Which of the following findings should the nurse identify as an adverse effect of the medication?

Cardiac dysrhythmias

A nurse is providing discharge teaching to a client who has venous thrombosis and a prescription for warfarin. Which of the following instructions should the nurse include in the teaching?

Carry a medical alert ID card

A nurse is caring for a client who has a new diagnosis of RA. The nurse should anticipate a prescription from the provider for which of the following medications for daily management of this condition?

Celecoxib

A nurse is preparing to teach a client who is to start a new prescription for extended release verapamil. Which of the following instruction should the nurse plan to include?

Change positions slowly. The nurse should instruct the client to change positions gradually to prevent orthostatic hypotension and syncope.

A nurse is checking a client who is receiving an IV infusion of telavancin for streptococcus pyogenes. Which of the following actions should the nurse include?

Check the client for pruritus

A nurse is caring for a client who has congestive HF and is taking digoxin. The client reports nausea and refuses to eat breakfast. Which of the following actions should the nurse take first?

Check the client's apical pulse

A nurse is providing teaching to a client who has hypothyroidism and is taking levothyroxine. The nurse should instruct the client that which of the following findings is an indication of thyrotoxicosis?

Chest pain

A nurse is caring for a client who is receiving sumatriptan for cluster headaches. Which of the following findings should the nurse expect as an adverse effect?

Chest pressure

A nurse is providing teaching to a client who is start taking sumatriptan. Which of the following adverse effects should the nurse instruct the client to monitor for and report to the provider?

Chest pressure Sumatriptan is an antimigraine agent which can cause coronary vasospasms, resulting in angina. The client should report chest pressure or heavy arms to the provider.

A nurse is preparing to administer the influenza vaccine to a client. Which of the following allergies should the nurse identify as a contraindication to the client receiving this vaccine?

Chicken eggs

A nurse is teaching an assistive personnel about dietary restrictions for a client who is taking phenelzine to treat depression The AP's selection of which of the following foods for the client's lunch indicates an understanding of the teaching?

Chicken salad

A nurse is caring for a client who has alcohol use disorder and was admitted with lower extremity fx following a MVA. A few hours after admission, the cllient develops restlessness and tremors. Which of the following medications should the nurse anticipate administering the client first?

Chlordiazepoxide

A nurse is caring for a client who is experiencing acute alcohol withdrawal. The nurse should expect to administer which of the following medications?

Chlordiazepoxide

A nurse is completing the admission history for a client who reports drinking 1 pint of whiskey every day for 6 years. The client's last drink was 10 hours ago. Which of the following medications should the nurse plan to administer upon admission?

Chlordiazepoxide

A nurse is reviewing the medical record of a client who might have hearing loss. Which of the following data from the client's medical record should the nurse identify as a risk factor for hearing loss?

Chronic use of salicylates

A nurse is caring for a client who is experiencing an acute gout attack. The nurse should anticipate a prescription from the provider for which of the following medications?

Colchicine

A nurse is preparing to administer an epinephrine IV bolus to a client. Which of the following should the nurse verify before initiating the IV medication?

Concentration of the formulation

A nurse is caring for a client who has a new prescription for meperidine 500 mg PO q 4-6 hrs to manage pain. Which of the following actions should the nurse take?

Contact the provider for clarification of the prescription

A nurse is reviewing the laboratory results of a client who is taking tobramycin and notes that the medication's peak level is 7 mcg/mL. Which of the following actions should the nurse take?

Continue to administer the medication as prescribed

A nurse is caring for a client who has hyperlipidemia and is receiving simvastatin 40 mg PO daily. Which of the following items should the nurse remove from the client's breakfast tray before it is delivered to the room?

Grapefruit juice

A nurse is caring for a client who has a suspected adrenal insufficiency. Which of the following medications should the nurse anticipate the provider using to determine the presence of adrenal insufficiency?

Cosyntropin

A nurse is reviewing laboratory results for a client who is to receive a dose of ceftazidime via intermittent IV bolus. Which of the following laboratory findings is the priority for the nurse to report to the provider before administering the medication?

Creatinine 2.6 mg/dL Ceftazidime is excreted primarily by the renal system. A serum creatinine level above 1.3 mg/dL can indicate a kidney disorder requiring a reduction in the dose administered. The nurse should notify the provider, who is likely to prescribe a lowered dose of medication.

A nurse is caring for a client who has alzheimer's disease and a prescription for memantine. Which of the following laboratory findings should the nurse identify as a contraindication to receiving this medication?

Creatinine clearance 35 mL/min

A nurse is caring for a client with Alzheimer's disease who has a new prescription for memantine. Which of the following laboratory results should the nurse identify as increasing the client's risk for decreased clearance of the medication?

Creatinine clearance 35 mL/min

A nurse is reviewing the laboratory data for a client who has alzheimer's disease and a new prescription for memantine. The nurse should identify that which of the following findings increases the client's risk for reduced clearance of this medication?

Creatinine clearance 35 mL/min

A nurse is caring for a client who has diabetes insipidus. Which of the following laboratory values should the nurse identify as reflecting a contraindication to receiving vasopressin to treat this disorder?

Creatinine clearance 50 mL/min

A nurse is caring for a client who is taking atorvastatin for hyperlipidemia. Which of the following client laboratory values should the nurse monitor?

Creatinine kinase MY ANSWER The client who is taking atorvastatin can develop an adverse effect called rhabdomyolysis, which causes muscle weakness or pain and can progress to myositis. Creatinine kinase levels rise in response to enzymes released with muscle injury.

A nurse is planning to teach about inhalant medications to a client who has a new diagnosis of exercise-induced asthma. Which of the following medications should the nurse plan to instruct the client to use prior to physical activity?

Cromolyn Cromolyn sodium stabilizes mast cells, which inhibit the release of histamine and other inflammatory mediators. The client should use cromolyn 10 to 15 min before planning to exercise to prevent bronchospasms.

A nurse is reviewing the medical record of a client who is requesting a prescription for sildenafil citrate. Which of the following data in the client's record should the nurse identify as a contraindication to the use of this medication?

Current use of isosorbide to treat HF

A nurse is caring for a client who takes GInkgo biloba daily at home. Which of the following effects should the nurse expect from the use of this herbal supplement?

Decreased platelet aggregation

A nurse is preparing to administer raloxifene to a client. Which of the following conditions is a contraindication for the administration of this medication?

DVT

A nurse is teaching a client who is premenopausal and has a prescription for a combination oral contraceptive. Which of the following findings should the nurse include as an adverse effect of oral contraceptives?

DVT

A nurse is administering baclofen for a client who has a spinal cord injury. Which of the following findings should the nurse document as a therapeutic outcome?

Decrease in flexor and extensor spasticity A client who has a spinal cord injury and takes baclofen can experience a decrease in the frequency and severity of muscle spasms and in flexor and extensor spasticity.

A nurse is caring for a client who has been taking metformin for 6 months. Which of the following findings should the nurse identify as an expected therapeutic effect of the medication?

Decreased blood glucose level

A nurse is assessing a client who has AIDS and is taking zidovudine. Which of the following findings is the priority for the nurse to report to the provider?

Decreased hemoglobin

A nurse is caring for a client who is taking budesonide to treat Crohn's disease. Which of the following findings should indicate to the nurse that the tx is effective?

Decreased inflammation

A nurse is administering brimonidine eye drops to a client who has glaucoma. Which of the following ocuar effects should the nurse expect?

Decreased intraocular pressure

A nurse is providing teaching to a client with TB who has prescriptions for rifampin and ethambuterol. Which of the following findings is an adverse effect of these medications that the client should report to the provider?

Decreased visual acuity

A nurse is assisting with a client's laceration repair in which the provider will use both lidocaine and epinephrine. The nurse should inform the client that the epinephrine will perform which of the following actions?

Delay systemic absorption of the anesthetic properties of lidocaine

A nurse is planning care for a client who has hypertension and is to start taking metoprolol. Which of the following interventions should the nurse include in the plan of care? Weigh the client weekly. Determine apical pulse prior to administering. Administer the medication 30 min prior to breakfast. Monitor the client for jaundice.

Determine apical pulse before admin Life-threatening bradycardia is an adverse effect that might affect this client. Therefore, the nurse should assess the client's apical pulse prior to administering the medication. If the client's pulse rate is less than 60/min, the nurse should withhold the medication and notify the provider.

A nurse is monitoring a client who has asthma, takes albuterol, and recently started taking propranolol to treat CVD. The client reports that the albuterol has been less effective. Which of the following factors should the nurse identify as a possible explanation for this change?

Detrimental inhibitory interaction

A nurse is reviewing the medical record of a client who is receiving HCTZ. The nurse should expect to find an improvement in which of the following conditions as a result of this medication?

Diabetes insipidus

A nurse is caring for a client who is taking acarbose to treat type 2 DM. For which of the following adverse effects of this medication should the nurse monitor the client?

Diarrhea

A nurse is providing discharge teaching to a client who is postop and has a new prescription for an oral opioid analgesic. Which of the following pieces of info should the nurse include as a rationale for increasing the client's daily intake of fiber?

Dietary fiber helps prevent constipation

A nurse is caring for a female client who has been taking clomiphene to treat infertility. Which of the following findings should indicate that the medication has been effective?

Follicular enlargement and conversion to corpus luteum after ovulation

A nurse is reviewing the medication administration record of a client who has hypocalcemia and a new prescription for IV calcium gluconate. The nurse should identify that which of the following medications can interact with calcium gluconate? Felodipine Guaifenesin Digoxin Regular insulin

Digoxin The nurse should identify that calcium gluconate can cause hypercalcemia, which increases the risk of digoxin toxicity.

A nurse is admitting a client who has afib with a HR of 155/min. The nurse should anticipate a prescription from the provider for which of the following medications?

Diltiazem

A nurse is caring for a client who has atrial fibrillation and is scheduled for cardioversion. The nurse should anticipate a prescription from the provider for which of the following medications for this procedure?

Diltiazem

A nurse is caring for a client who is receiving continuous cardiac monitoring. Which of the following medications should the nurse anticipate administering to treat atrial fibrillation?

Diltiazem

A nurse is administering cefotetan via intermittent IV bolus to a client who suddenly develops dyspnea and widespread hives. Which of the following actions should the nurse take first?

Discontinue the medication IV infusion. The greatest risk to the client is respiratory arrest from anaphylaxis. Therefore, the first action the nurse should take is to discontinue the medication IV infusion to prevent the client from receiving more medication. However, the nurse should not remove the IV catheter. Instead, the nurse should change the tubing and administer 0.9% sodium chloride by continuous IV infusion.

A nurse is caring for a client who has developed hypomagnesemia due to long-term therapy with lansoprazole. The nurse should monitor the client for which of the following manifestations?

Disorientation The nurse should monitor the client for disorientation and confusion as manifestations of hypomagnesemia. The nurse should also assess the client for a positive Chvostek's and Trousseau's signs.

A nurse is caring for a client who has acute glomerulonephritis and a prescription for furosemide. The nurse should monitor the client for which for the following therapeutic effects of this medication?

Diuresis

A nurse is caring for an older adult client who has a prescription for zolpidem at bedtime to promote sleep. The nurse should plan to monitor the client for which of the following adverse effects?

Dizziness

A nurse is preparing to administer amlodipine to a client who has hypertension. The nurse should plan to monitor the client for which of the following adverse effects of the medication?

Dizziness, palpitation, peripheral edema

A nurse is preparing to administer medications to a client who tells the nurse, "I don't want to take my fluid pill until I get home today." Which of the following actions should the nurse take? Document the refusal and inform the client's provider. File an incident report with the risk manager. Contact the pharmacist to pick up the medication. Give the client the medication to take at home and document that it was administered.

Document refusal and notify the provider The nurse has the responsibility to verify that the client understands the risks of refusing the medication so that an informed decision can be made. The nurse should then document the refusal in the client's medical record and notify the health care provider.

A nurse is teaching a client who has a new prescription for docusate sodium about the medication's mechanism of action. Which of the following information should the nurse include in the teaching?

Docusate sodium reduces the surface tension of the stools to change their consistency. Docusate sodium is a surfactant that softens stool by reducing surface tension, allowing water to penetrate more easily into the stool.

A nurse is caring for a client who has pneumonia. The client tells the nurse she is pregnant and that she has not told her provider yet. The nurse should identify that pregnancy is a contraindication for receiving which of the following medications?

Doxycycline Doxycycline is a tetracycline antibiotic. The nurse should identify that doxycycline can cause teratogenic effects such as staining of the infant's teeth when exposed to this medication. Therefore, this medication is contraindicated for the client.

A nurse is teaching a client who is taking allopurinol for the treatment of gout. Which of the following information should the nurse include in the teaching?

Drink 2L of water daily. The nurse should instruct the client to drink at least 2 L of water each day to prevent renal stone formation and kidney injury, because allopurinol is eliminated through the kidneys.

A nurse is providing teaching to a client who has a prescription for trimethoprim/sulfamethoxazole. Which of the following instructions should the nurse include in the teaching?

Drink 8 to 10 glasses of water daily. The nurse should instruct the client to increase water intake to 1,920 to 2,400 mL (65 to 81 oz) a day to decrease the chance of kidney damage from crystallization.

A nurse is caring for a client who is taking selegiline. The nurse should monitor the client for which of the following adverse effects of selegiline and notify the provider if it occurs?

Drowsiness

A nurse is administering oral hydroxyzine to a client. Which of the following adverse effects should the nurse instruct the client to expect?

Dry mouth

A nurse is assessing a client who is taking amitriptyline for depression. Which of the following findings should the nurse identify as an adverse effect of the medication?

Dry mouth The nurse should expect the client to have a dry mouth due to the blocking of acetylcholine receptors that cause anticholinergic responses.

A nurse is caring for a client who has asthma and advanced RA and deformity of the hands. The nurse should anticipate that the client will receive which of the following medication delivery devices for the tx of asthma?

Dry powder inhaler

A nurse is administering adenosine via IV bolus for a client who has developed paroxysmal atrial tachycardia. For which of the following findings should the nurse assess the client during the administration of adenosine?

Dyspnea

A nurse is caring for an older adult client who has a new prescription for amitriptyline to treat depression. Which of the following diagnostic tests should the nurse plan to perform prior to staring the client on this medication?

Electrocardiogram

A nurse is caring for a male client who has been taking cimetidine for the tx of a duodenal ulcer. Which of the following manifestations related to the medication should the nurse report to the provider?

Emesis that looks like coffee grounds

A nurse is caring for a client who is 12 hours postop following a total hip arthroplasy. Which of the following medications should the nurse anticipate administering to this client to prevent DVT?

Enoxaparin

A nurse is administering diazepam to a client who is having a colonoscopy. Which of the following actions should the nurse take?

Ensure flumazenil is available to administer for toxicity management. The nurse should monitor the client for manifestations of diazepam toxicity, such as respiratory depression and hypotension. The nurse should be prepared to administer flumazenil to reverse the effects of diazepam.

A nurse is assessing a client after administering a second dose of cefazolin IV. The nurse notes the client has anxiety, hypotension, and dyspnea. Which of the following medications should the nurse administer first? Diphenhydramine Albuterol inhaler Epinephrine Prednisone

Epinephrine According to evidence-based practice, the nurse should administer epinephrine first to induce vasoconstriction and bronchodilation during anaphylaxis.

A nurse is caring for a client who is receiving cefotetan 1 g via intermittent IV bolus every 12 hr to treat a postop infection. Which of the following manifestations should the nurse monitor for as an adverse effect of the medication?

Epistaxis

A nurse is caring for a client who has a vit K deficiency. Which of the following manifestations should the nurse expect?

Excessive bruising

A nurse is caring for a client who takes scheduled morphine for CA pain. The client reports experiencing breakthrough pain. The nurse should anticipate a prescription from the provider for which of the following medications to treat breakthrough pain?

Fentanyl

A nurse is providing teaching to a client who has gout and a new prescription for allopurinol. The nurse should instruct the client to discontinue taking the medication for which of the following adverse effects?

Fever

A nurse in an emergency department is caring for a client whose family reports the client has taken large amounts of diazepam. Which of the following medications should the nurse anticipate administering? Ondansetron Magnesium sulfate Flumazenil Protamine sulfate

Flumazenil The nurse should anticipate administering flumazenil, an antidote used to reverse benzodiazepines such as diazepam.

A nurse is providing teaching to a client who has peptic ulcer disease and is to start a new prescription for sucralfate. Which of the following actions of sucralfate should the nurse include in the teaching? Decreases stomach acid secretion Neutralizes acids in the stomach Forms a protective barrier over ulcers Treats ulcers by eradicating H. pylori

Forms a protective barrier over ulcers Secretions by the parietal and chief cells, hydrochloric acid and pepsin, can further irritate the ulcerated areas. Sucralfate, a mucosal protectant, forms a gel-like substance that coats the ulcer, creating a barrier to hydrochloric acid and pepsin.

A nurse is caring for a client who is taking streptomycin. Which of the following medications increases the client's risk of developing ototoxicity when takken with streptomycin?

Furosemide

A nurse is performing a preop assessment of a client who is about to undergo an aneurysm clipping. The nurse should identify a risk for increased bleeding when the client reports taking which of the following dietary supplements?

Garlic

A nurse is teaching a client who has severe generalized RA and is schedule to start taking prednisone for long term therapy. The nurse should instruct the client to report which of the following as an adverse effect of prednisone?

Gastric ulceration

A nurse is preparing to administer the varicella vaccine to a 12 month old infant. The nurse asks the infant's guardian if the infant has any allergies. Which of the following allergies is a contraindication to the infant receiving the vaccine?

Gelatin

A nurse is assessing a client's vital signs prior to the administration of PO digoxin. The client's BP is 144/86 mm Hg, heart rate is 55/min, and respiratory rate is 20/min. The nurse should withhold the medication and contact the provider for which of the following findings? Diastolic BP Systolic BP Heart rate Respiratory rate

HR Digoxin slows the conduction rate through the SA and AV nodes, thereby decreasing the heart rate. The nurse should withhold the medication and notify the provider for a heart rate of 55/min because this is an early indication of digoxin toxicity.

A nurse is preparing to administer digoxin to a client. Which of the following findings should the nurse identify as a contraindication to the client receiving this medication?

HR 51/min

A nurse is teaching a client who has chemotherapy induced anemia and a prescription for epoetin alfa. The nurse should instruct the client to report which of the following findings as an adverse effect of epoetin alpha?

HTN

A nurse contacts a client's provider on the telephone to obtain a prescription for pain medication. Which of the following actions should the nurse take?

Have the provider spell out the unfamiliar medication names. The nurse should ask the provider to spell out the name of the medication if the stated name is one the nurse is not familiar with.

A nurse is teaching a newly licensed nurse about caring for a client who is receiving PCA. Which of the following actions by the new nurse indicates an understanding of the teaching?

Having a second nurse check the PCA setting

A nurse is monitoring a client who has diabetes insipidus and was administered desmopressin. Which of the following findings should indicate to the nurse the client is experiencing an adverse effect of this medication?

Headache

A nurse is caring for a client who has a new prescription for levothyroxine to treat hypothyroidism. Which of the following findings should the nurse identify as an indication that the client requires intervention?

Heart rate 106/min

A nurse is reviewing the medical record of a client who has postmenopausal osteoporosis and a prescription for raloxifene. Which of the following findings in the client's medical record should the nurse identify as a contraindication to receiving this medication?

Hx of DVT

A nurse is reviewing the laboratory results of a client who is taking a medication and notes that the client's blood tests show an elevated level of the enzymes AST and ALT. The nurse should recognize that these findings are potential indicators of which of the following conditions?

Hepatic toxicity

A nurse is reviewing the medical history of a client who has spasticity due to MS and a new prescription for tizanidine. Which of the following comorbidities increases the client's risk of adverse effects while taking this medication?

Hepatitis

A nurse is reviewing the medical record of a client with RA who has a prescription for infliximab. Which of the following findings should the nurse identify as a contraindication to the client receiving this medication?

Hepatitis B virus

A nurse in a provider's office is reviewing a client's medication history. The client asks the nurse if she should begin taking high dose vitamins as she ages. Which of the following pieces of info should the nurse provide about high doses of vit supplements?

High dose of water soluble vitamins can have adverse effects

A nurse is preparing to administer dantrolene to a client who has muscle spasticity. Which of the following findings form the client's medical history should the nurse identify as a contraindication to the administration of this medication?

History of cirrhosis

A nurse is caring for a client who has osteoporosis and has been taking a vit D supplement. The nurse notes that the client reports also taking a multivitamin daily. Which of the following findings should indicate to the nurse that the client might be experiencing vit D toxicicty?

Hypercalcemia

A nurse is caring for a client who has heart failure and a prescription for enalapril. The nurse should monitor the client for which of the following findings as an adverse effect of the medication?

Hyperkalemia Enalapril improves cardiac functioning in clients who have heart failure and can cause hyperkalemia due to potassium retention by the kidneys.

A nurse in a provider's office is assessing a client who has been taking aspirin daily for the past year. For which of the following findings should the nurse notify the provider immediately?

Hyperventilation When using the urgent vs. nonurgent approach to client care, the nurse should determine that the priority finding is hyperventilation. This finding indicates the client might have acute salicylate poisoning, which causes respiratory alkalosis in the early stages.

A nurse is caring for a client who is taking fludrocortisone. Which of the following findings indicates to the nurse that the client is experiencing an adverse effect of the medication?

Hypokalemia

A nurse is caring for a client who has a new prescription for enalapril. The nurse should monitor the client for which of the following adverse effects of this medication?

Hypotension

A nurse is caring for a client with BPH who has a new prescription for doxazosin. Which of the following manifestations should the nurse monitor for as an adverse effect of doxazosin?

Hypotension

A nurse is planning to administer diltiazem via IV bolus to a client who has afib. When assessing the client, the nurse should recognize that which of the following findings is a contraindication to administration of diltiazem?

Hypotension

A nurse is preparing to administer a SL nitroglycerin tab to a client who is reporting chest pain. For which of the following adverse effects should the nurse monitor after giving this medication?

Hypotension

A nurse is teaching a group of nurses about the effects of a client receiving spinal anesthesia. Which of the following pieces of info should the nurse include in the teaching?

Hypotension is an adverse effect of spinal anesthesia

A nurse is teaching a client who is to start taking ranitidine for peptic ulcer disease. Which of the following client statements should the nurse identify as understand of the teaching?

I know smoking makes ranitidine less effective. The nurse should instruct the client that smoking decreases the effectiveness of ranitidine by exacerbating the ulcer manifestations.

A nurse is teaching a client about the use of risedronate for the treatment of osteoporosis. The nurse should identify which of the following statements as an indication that the client understands the teaching? "I will drink a glass of milk when I take the risedronate." "I will take the risedronate 15 minutes after my evening meal." "I should take an antacid with the risedronate to avoid nausea." "I should sit up for 30 minutes after taking the risedronate."

I should sit up for 30 min after taking risedronate Sitting upright for at least 30 min after taking risedronate will reduce the adverse gastrointestinal effects of esophagitis and dyspepsia. Risedronate is contraindicated for a client who cannot sit or stand upright for this length of time.

A nurse is providing discharge instructions to a client who has heart failure and a new prescription for captopril. Which of the following client statements indicates an understanding of the teaching? "I should take the medication with food." "I should take naproxen if I develop joint pain." "I should tell my provider if I develop a sore throat." "I should expect the medication to cause my urine to look orange."

I should tell my provider if I develop a sore throat The client should report a sore throat to the provider because this can indicate neutropenia, a serious adverse effect of captopril. Neutropenia can be reversed if it is identified early and the medication is promptly discontinued.

A nurse is providing teaching to a client about the use of ethinyl estradiol/norelgestromin. The nurse should identify that which of the following statements by the client indicates an understanding of the teaching?

I will fold the sticky side of the old patch together before disposing. The client should fold the sticky sides of the old patch together and then place it in a childproof container to ensure safe disposal of the patch.

A nurse is providing discharge instructions to a client who is to self-administer insulin at home. Which of the following client statements should indicate to the nurse that the teaching is effective?

I will store my unopened bottles of insulin in the refrigerator. The client should store unopened vials of insulin in the refrigerator to maintain medication viability. Once opened, the insulin can remain at room temperature for up to 1 month.

A nurse is teaching about zolpidem to a client who has insomnia. The nurse should identify that which of the following client statements indicates an understanding of the teaching?

I will use this medication for a short period of time Zolpidem is used for short-term treatment of insomnia. Therefore, the provider should reassess the client before refilling the prescription.

A nurse is caring for a client who is taking warfarin. Which of the following laboratory values should the nurse recognize as an effective response to the medication?

INR 3.0

A nurse is reviewing the lab reports of a client who has been taking warfarin for afib. Which of the following results should the nurse report to the provider immediately?

INR 5.5

A nurse is reviewing the medical record for a client who has a migraine and a prescription for sumatriptan. Which of the following factors in the client's medical hx should the nurse identify as a contraindication to receiving this med?

Ischemic heart disease

A nurse is caring for a client who has HF and is prescribed dobutamine hydrochloride by continuous IV infusion. The nurse should identify that which of the following is the therapeutic effect of this medication?

Improves cardiac output

A nurse is providing discharge teaching to a client who has a new prescription for furosemide twice daily. The nurse should include which of the following instructions in the teaching?

Inc intake of potassium rich food Loop diuretics, such as furosemide, act at the loop of Henle by blocking the resorption of sodium, water, and potassium. An adverse effect of the medication is the development of electrolyte imbalances such as hyponatremia, hypochloremia, and hypokalemia. To prevent hypokalemia, the client should increase intake of potassium-rich foods, such as potatoes, spinach, dried fruit, and nuts. Monitor for muscle weakness Furosemide, a loop diuretic, causes a loss of potassium, which can result in manifestations of hypokalemia such as difficulty concentrating, shallow respirations, hyporeflexia, and muscle weakness. The nurse should instruct the client to monitor for these manifestations and report them to the provider. Dangle your legs from the side of the bed before standing Loop diuretics, such as furosemide, reduce vascular tone and increase fluid excretion. These effects decrease blood return to the heart and can manifest as dizziness and lightheadedness when going from a lying to a standing position. The client should change positions slowly to minimize orthostatic hypotension.

A nurse is preparing to administer oxytocin to a client who is at 41 weeks gestation and is experiencing ineffective labor. Which of the following actions should the nurse plan to take?

Increase the dose of oxytocin to obtain uterine contractions that occur every 2-3 min

A nurse is assessing a client who is receiving epoetin alfa to treat anemia. Which of the following findings should the nurse monitor?

Increased blood pressure. The therapeutic effect of epoetin alfa is an increase in hematocrit levels, which can result in an increase in a client's blood pressure. If the client's hematocrit level rises too rapidly, hypertension and seizures can result. The nurse should monitor the client's blood pressure and ensure hypertension is controlled prior to administering the medication.

A nurse is caring for a client who has heart failure and is receiving an IV infusion of dopamine. Which of the following findings indicated that the medication is effective?

Increased cardiac output. Dopamine is an adrenergic that causes a receptor specificity effect, which increases cardiac output and improves perfusion.

A nurse is teaching a client who had kidney transplant surgery about immunosuppressive medications. Which of the following adverse effects of these medications should the nurse include in the teaching?

Increases susceptibility to infection

A nurse is planning care for a client with thrombophlebitis who has a prescription to receive heparin via continuous IV infusion. Which of the following actions should the nurse include in the plan of care?

Infuse the heparin using an electronic IV pump

A nurse is preparing a continuous IV infusion of erythromycin lactobionate for a client who has a Bordetella pertussis infection. Which of the following actions should the nurse take to minimize the risk of thrombophelebitis?

Infuse the medication slowly

A nurse is caring for a client who has a prescription for an oral contraceptive to prevent pregnancy. The nurse should identify that which of the following actions is the purpose of this medication?

Inhibition of ovulation

A nurse is providing teaching to a newly licensed nurse about metoclopramide. The nurse should highlight that which of the following conditions is a contraindication to this medication?

Intestinal obstruction

A nurse is caring for a client who is in preterm labor and has a new prescription for nifedipine. The client states she is concerned becuase her father takes nifedipine for his angina pectoris. The nurse should explain that nifedipine works for clients who are pregnant through which of the following mechanisms?

It inhibits uterine contractions by blocking the entry of calcium into uterine cells

A charge nurse is teaching a newly licensed nurse about a client who has severe allergy related asthma and a new prescription for omalizumab. Which of the following pieces of info should the charge nurse include to describe the medication's mechanism of action?

It reduces the number of imunoglobulin E molecules on mast cells.

A nurse is providing teaching to a client who has ulcerative colitis and a new prescription for sulfasalazine. The nurse should instruct the client to monitor for which of the following adverse effects of this medication?

Jaundice

A nurse is monitoring a client with pneumonia who has received penicillin G IM. Which of the following findings should the nurse plan to evaluate first?

Laryngeal edema

A nurse is assessing a client who is taking tamoxifen to treat breast cancer. Which of the following findings is the priority for the nurse report to the provider?

Leg tenderness The greatest risk to this client is the development of a thromboembolism, which is an adverse effect of tamoxifen. The nurse should also monitor the client for other manifestations of a thromboembolism, including leg tenderness, redness, swelling, and shortness of breath.

A nurse is reviewing laboratory values for a client who reports fatigue and cold intolerance. The client has an increased TSH level and a decreased total T3 and T4 level. The nurse should anticipate a prescription for which of the following medications?

Levothyroxine

A nurse is reviewing the medication administration record of a client who has impaired swallowing. The nurse should crush the medication when administering which of the following prescriptions?

Levothyroxine 75 mcg PO q AM before breakfast

A nurse is administering a medication to a client. The nurse should identify that which of the following medication distribution factors facilitates the effective passage of the medication across the client's cell membranes?

Lipid solubility

A nurse is reviewing the medical record of a client who has a prescription for a combination oral contraceptive. The nurse should identify that which of the following findings is a contraindication to receiving this medication?

Liver disease

A nurse is providing teaching to a newly licensed nurse about caring for a client who has a prescription for gemfibrozil. The nurse should instruct the newly licensed nurse to monitor which of the following laboratory tests?

Liver function

A nurse is reviewing the MAR of a client who is receiving an opioid medication for pain. Which of the following prescriptions should the nurse clarify with the provider?

Lorazepam

A nurse is assessing a client who has multidrug resistant TB and takes ethambutol. The nurse should identify which of the following findings as an adverse effect of this medication?

Loss of red/green color discrimination

A nurse is reviewing the medical record of a client who has been taking a vit D supplement. Which of the following findings from the cient's record should the nurse identify as a risk factor for developing vit D deficiency?

Obesity

A charge nurse is monitoring a newly licensed nurse who is caring for a postoperative client who is receiving morphine through a PCA pump. Which of the following actions by the newly licensed nurse requires intervention?

Maintaining the client on bed rest while the PCA pump is in use.

A nurse is monitoring a client who is receiving terbutaline to suppress preterm labor. Which of the following findings should indicate to the nurse that the client is experiencing an adverse effect of the medication?

Maternal heart rate > 120/min

A nurse working in a mental health facility is admitting a client with opioid use disorder who is experiencing withdrawal. The nurse should anticipate a prescription for which of the following medications from the provider?

Methadone

A nurse is caring for a client who is to receive treatment for opioid use disorder. Which of the following medications should the nurse expect to administer?

Methadone The nurse should expect to administer methadone for treatment of opioid use disorder. Methadone can be administered for withdrawal and to assist with maintenance and suppressive therapy.

A nurse is caring for a client who takes a combination oral contraceptive. Which of the following findings should indicate to the nurse that the client is experiencing a deficiency of estrogen in the OC?

Mid cycle breakthrough bleeding or spotting

A nurse is caring for a client who has a new prescription for ergotamine. The nurse should recognize that ergotamine is administered to treat which of the following conditions?

Migraine HA

A nurse is caring for a client who was recently diagnosed with Addison's disease and has been placed on long term mineralcorticoid therapy with fludrocortisone. Which of the following pieces of info shoulld the nurse provide when explaining the purpose of this therapy?

Mineralocorticoids maintain electrolyte and fluid balance

A nurse is assessing a client who has myasthenia gravis and is taking neostigmine. Which of the following findings should indicate to the nurse that the client is experiencing an adverse effect? Tachycardia Oliguria Xerostomia Miosis

Miosis Miosis, which is pupillary constriction, is a common adverse effect of neostigmine due to the excessive muscarinic stimulation that causes difficulty with visual accommodation.

A nurse is reinforcing teaching with a newly licensed nurse about contraindications to vaccines. Which of the following examples should the nurse provide as a true contraindication for all vaccines?

Moderate illness without a fever

A nurse is preparing to administer PO sodium polystyrene sulfonate to a client who has hyperkalemia. Which of the following actions should the nurse plan to take? Hold the client's other oral medications for 8 hr post administration. Inform the client that this medication can turn stool a light tan color. Keep the client's solution in the refrigerator for up to 72 hr. Monitor the client for constipation.

Monitor for constipation The nurse should monitor the client for the adverse effect of constipation and report it to the provider because this can lead to fecal impaction.

A nurse is assessing a client how is taking varenicline for smoking cessation. Which of the following findings is the nurse's priority?

Mood changes

A nurse receives a verbal order from the provider to administer morphine five milligrams every 4 hours subcutaneously for severe pain as needed. The nurse should identify which of the following entries as the correct format for the medication administration record (MAR)? MSO4 5 mg subcut every 4 hr PRN severe pain Morphine 5 mg subcut every 4 hr PRN severe pain MSO4 5 mg SQ every 4 hr PRN severe pain Morphine 5.0 mg subcutaneously every 4 hr PRN severe pain

Morphine 5 mg subcut every 4 hr PRN severe pain The nurse should identify this entry as the correct format for the MAR. The medication name is spelled out and there are not any abbreviations from The Joint Commission's "Do Not Use" list included in the transcription.

A nurse is caring for a client who has a prescription for chlorthiazide to treat HTN. The nurse should plan to monitor the client for which of the following adverse effects?

Muscle weakness

A nurse is providing teaching to a client who has type 2 DM and a new prescription for metformin. Which of the following adverse effects of metformin should the nurse instruct the client to watch for and report to the provider?

Myalgia

A nurse is caring for a client who has been in the PACU for more than 1 hr, has a RR of 9/min, and is difficult to arouse. The nurse should expect a prescription for which of the following medications?

Naloxone

A nurse in an emergency department is caring for a client who has heroin toxicity. The client is unresponsive with pinpoint pupils and a respiratory rate of 6/min. Which of the following medications should the nurse plan to administer? Methadone Naloxone Diazepam Bupropion

Naloxone The nurse should administer naloxone, an opioid antagonist, to a client who has heroin toxicity to reverse the respiratory depressive effects of the heroin. However, the nurse should not administer naloxone too quickly because naloxone can cause hypertension, tachycardia, nausea, vomiting, and might cause the client to enter a state of opioid withdrawal.

A nurse is caring for a client who is experiencing an acute gout attack. The nurse should anticipate a prescription from the provider for which of the following medications?

Naproxen

A nurse is caring for a client who has a new prescription for amphotericin B. The nurse should plan to monitor the client for which of the following adverse effects?

Nephrotoxicity

A nurse is reviewing the lab results for a client who has a prescription for filgrastim. An increase in which of the following values indicates therapeutic effect of this medication?

Neutrophil count

A nurse is caring for a client who is at 28 weeks gestation and is experiencing preterm labor. Which of the following medications should the nurse plan to administer?

Nifedipine

A nurse is caring for a client who reports lethargy and myalgia after taking clozapine for 6 months. Which of the following actions should the nurse plan to take?

Obtain WBC with absolute neutrophil count. The client who takes clozapine can develop lethargy and myalgia caused by the adverse effect of agranulocytosis. Therefore, monitoring the WBC with absolute neutrophil count weekly for the first 6 months of treatment is recommended. After 6 months, monitoring can occur every 2 weeks up to 1 year.

A nurse is admitting a client who has unstable angina. Which of the following medications should the nurse anticipate administering to the client?

Nitroglycerin

A nurse is caring for a client who has unstable angina. The nurse should anticipate a prescription from the provider for which of the following medications?

Nitroglycerin

A nurse is caring for a client who reports crushing chest pain. The nurse reviews the client's ECG results and notes ST changes. Which of the following medications should the nurse administer?

Nitroglycerin

A nurse administered digoxin immune Fab to a client who received the incorrect dose of digoxin over a period of 3 days. The nurse should identify that which of the following findings indicates the antidote was effective?

Normal sinus rhythm Dysrhythmias are a life-threatening adverse effect of digoxin toxicity. The return of the heart to normal sinus rhythm indicates a therapeutic response to the antidote. Digoxin immune Fab is administered to a client who is experiencing severe digoxin toxicity. It binds with digoxin and works to reduce the client's blood digoxin level.

A nurse is providing teaching to a client with asthma who has a new prescription for a short-acting beta-2 agonist (SABA) bronchodilator. Which of the following pieces of info should the nurse share?

Notify the provider if the SABA is needed more than twice per week.

A nurse is administering a prescription for nifedipine to a client who is pregnant. Which of the following pieces of info should the nurse monitor and document?

Number of uterine contractions

A nurse is preparing to administer hydrocholorothiazide (HCTZ) to a client. Which of the following actions should the nurse take prior to administer the medication?

Obtain the client's blood pressure HCTZ is a thiazide diuretic administered to promote urine output and reduce blood pressure and edema. The nurse should obtain the client's blood pressure prior to administration of the medication.

A nurse is preparing to administer medication to a client who has gout. The nurse discovers that an error was made during the previous shift and the client received atenolol instead of allopurinol. Which of the following actions should the nurse take first? Obtain the client's blood pressure. Contact the client's provider. Inform the charge nurse. Complete an incident report.

Obtain the clients BP When using the nursing process, the first action the nurse should take to prevent injury to the client is to assess the client for adverse effects of atenolol, such as hypotension.

A nurse is planning to teach about the use of a spacer to a child who has a new prescription for a fluticasone inhaler to treat chronic asthma. The nurse should include that the spacer decreases the risk for which of the following adverse effects of the medication? Oral candidiasis Headache Joint pain Adrenal suppression

Oral candidiasis Dysphonia and oral candidiasis are adverse effects of inhaled corticosteroids. Using a spacer and rinsing the mouth after inhalation will minimize the amount of medication remaining in the oropharynx, preventing the development of these adverse effects.

A nurse is teaching the parent of a child who has severe reactive airway disease about glucocorticoid therapy. The parent asks why her child has to inhale the medication instead of taking it orally. Which of the following pieces of info should the nurse provide to the parent?

Oral glucocorticoids are more likely to slow linear growth in children

A hospice nurse is caring for a client who has CA and is taking naproxen 250 mg TID PO and gabapentin 1800 mg TID to manage pain. The client tells the nurse, "I'm having pain that keeps me from doing what I'd like most of the time." Which of the following additions should the nurse anticipate to the client's medication regimen?

Oral oxycodone

A nurse is caring for a client who has schizophrenia and a prescription for chlorpromazine. For which of the following adverse effects should the nurse monitor?

Orthostatic hypotension

A nurse is monitoring for adverse effects of hydrochlorothiazide after administering the medication to an older adult client who has heart failure. Which of the following findings should the nurse identify as an adverse effect of the medication?

Orthostatic hypotension The nurse should identify that hydrochlorothiazide is an antihypertensive thiazide diuretic medication, which can cause orthostatic hypotension and light headedness. Therefore, the nurse should instruct the client to rise slowly when moving from a recumbent to a standing position.

A nurse is caring for a client who is taking a prescription for glucocorticoid adrenal replacement medication for the long-term tx of Addison's disease. Which of the following findings indicates that the client is experiencing an adverse effect of the medication?

Osteoporosis

A nurse is reviewing the laboratory results for a client who is receiving heparin via continuous IV infusion for deep vein thrombosis. The nurse should discontinue the medication infusion for which of the following client findings? Potassium 5.0 mEq/ L aPTT 2 times the control Hemoglobin 15 g/dL Platelets 96,000/mm3

PLT 96,000 A platelet count of 96,000/mm3 is below the expected range of 150,000 to 400,000/mm3. A platelet count less than 100,000/mm3 while receiving heparin can indicate heparin-induced thrombocytopenia, a potentially fatal condition that requires stopping the infusion.

A nurse is planning care for a client who has a seizure disorder and a new prescription for valproic acid. Which of the following laboratory values should the nurse plan to monitor?

PTT, AST, ALT

A nurse is teaching a client who has ADHD and is starting therapy with an amphetamine/dextroamphetamine mixture. Which of the following manifestations should the nurse instruct the client to identify as an adverse effect and report to the provider?

Palpitations

A nurse is assessing a client who was recently admitted and has a hx of alcohol use disorder. The client display ataxia, and altered LOC, and nystagmus. Which of the following medications should the nurse anticipate administering to the client?

Parenteral thiamine

A nurse is assessing a child who has acute lymphocytic leukemia and is receiving vincristine sulfate. Which of the following findings is the nurse's priority?

Paresthesia

The nurse is assessing a client who has been taking linezolid to treat a staph aureus infection. Which of the following findings should the nurse report to the provider?

Paresthesias

A nurse is reviewing the medical record of a client who is scheduled for induction of labor and has a prescription for misoprostol. Which of the following conditions should the nurse identify as a contraindication to administering this medication?

Past cesarean delivery

A nurse is caring for a client who is taking glucocorticoids. The nurse should monitor the client for which of the following adverse effects of the medication?

Peptic ulcer

A nurse is caring for a client who has diabetes mellitus and is taking glyburide. The client reports feeling confused and anxious. Which of the following actions should the nurse take first?

Perform a capillary blood glucose test. The greatest risk to this client is injury from hypoglycemia. Therefore, the nurse should perform a capillary blood glucose test to determine the client's blood glucose status. Manifestations of hypoglycemia include weakness, anxiety, confusion, sweating, and seizures.

A nurse is providing teaching to a client who has a new prescription for doxycycline. The nurse should instruct the client to monitor for which of the following adverse effects?

Photosensitivity

A nurse is caring for a client who was brought to the ED by friends after a reported heroin overdose. Which of the following findings should the nurse expect to assess?

Pinpoint pupils

A nurse is teaching a newly licensed nurse about contraindicaton to ceftriaxone. A severe allergy to which of the following medications is a contraindication to ceftriaxone?

Piperacillin

A circulating nurse is planning care for a client who is scheduled for surgery and has a latex allergy. Which of the following actions should the nurse include in the plan of care?

Place monitoring cords and tubes in a stockinet. The nurse should place monitoring devices in a stockinet to prevent direct contact with the client's skin.

A nurse is caring for a client who received spinal anesthesia 30 minutes ago. The client reports feeling dizzy, and the nurse notes that the client's BP is 84/54 mmHg. Which of the following actions should the nurse take?

Place the client in the head down position

A nurse is caring for a client who has been receiving medication through a transdermal patch. The client is experiencing therapeutic benefits from the medication even though the medication in the patch is no longer active. The nurse should recognize that this is an example of which of the following?

Placebo effect

A nurse is teaching a client who has type 1 DM about a new subq insulin infusion pump. Which of the following pieces of info should the nurse include in the teaching?

Plan to use a type of short duration insulin in the infusion pump

A nurse is monitoring the laboratory values of a male client who has leukemia and is receiving weekly chemotherapy with methotrexate via IV infusion. Which of the following laboratory values should the nurse report to the provider?

Platelets 78000/mm^3

A nurse is reviewing the laboratory results of a client who is taking digoxin for heart failure. Which of the following results should the nurse report to the provider? Calcium level 9.2 mg/dL Magnesium level 1.6 mEq/L Digoxin level 1.1 ng/mL Potassium level 2.8 mEq/L

Potassium 2.8 A potassium level of 2.8 mEq/L is below the expected reference range of 3.5 to 5 mEq/L. The nurse should notify the provider if a client has hypokalemia prior to administration of digoxin due to the increased risk of developing digoxin toxicity and cardiac dysrhythmias.

A nurse is providing teaching to a female client who has a prescription for pravastatin to treat hyperlipidemia. Which of the following pieces of info should the nurse include in the teaching?

Pravastatin can be taken with grapefruit juice

A nurse is assigned to care for several clients who are postoperative. The client taking with of the following medications is at risk of delayed wound healing?

Prednisone to treat persistent arthritis exacerbations

A nurse is caring for a 20 year old female client who has a prescription for isotretinoin for severe nodulocystic acne vulgaris. Before the client can obtain a refill, the nurse should advise the client that which of the following tests is required? Serum calcium Pregnancy test 24-hr urine collection for protein Aspartate aminotransferase level

Pregnancy Test

A nurse is caring for a client at 39 weeks of gestation who has gestational HTN. The client has a new prescription for misoprostol for cervical ripening and induction of labor. Which of the following findings in the client's medical history should the nurse identify as increasing the client's risk of complications due to the use of this medication?=

Previous cesarean delivery

A nurse is providing teaching to a client who has chronic kidney failure with an AV fistula for hemodialysis and a new prescription for epoetin alfa. Which of the following therapeutic effects of epoetin alfa should the nurse include in the teaching?

Promotes RBC production

A nurse is caring for a client who is experiencing an acute asthma exacerbation. Which of the following medications should the nurse identify as being contraindicated for this client?

Propranolol

A nurse is caring for a client who is at 6 weeks of gestation and has just received a diagnosis of hyperthyroidism. The nurse should anticipate a prescription from the provider for which of the following medications?

Propylthiouracil

A nurse is assessing a client who is receiving IV gentamicin 3 times daily. Which of the following findings indicates that the client is experiencing an adverse effect of this medication?

Proteinuria

A nurse in an acute care facility is preparing a reconciled list of medications for a client who is being discharged home. Which of the following actions should the nurse take?

Provide the client and the next care provider with a list of medications the client will take after discharge

A nurse is preparing to administer an otic medication to an adult client. Which of the following actions should the nurse take?

Pull the pinna of the client's ear upward and outward.

A nurse is caring for a client who is receiving bleomycin IV to treat lymphoma. Which of the following assessments is the nurse's priority?

Pulmonary function

A nurse is preparing to administer medication to a client. The nurse should understand that which of the following abbreviations indicates the greatest frequency of medication administration?

QID

A nurse is providing teaching to the parents of a child who has a new prescription for lamotrigine for a seizure disorder. The nurse should instruct the parents that which of the following adverse effects is the priority to report to the provider?

Rash

A nurse is caring for a client who has asthma and requires long-term treatment. The nurse should identify that which of the following medications used for long-term treatment places the client at an increased risk of asthma-related death?

Salmeterol

A nurse is reviewing the medical record of a client. The medication administration record shows the client is taking clopidogrel. Which of the following events should the nurse expect in the client's medical history?

Recent MI

A nurse is preparing to mix and administer dantrolene via IV bolus to a client who has developed malignant hyperthermia during surgery. Which of the following actions should the nurse take?

Reconstitute the initial dose with 60mL of sterile water w/o a bacteriostatic agent. The nurse should dilute the medication with 60 mL of sterile water without a bacteriostatic agent and inject rapidly.

A nurse is caring for a client who has TB and is taking rifampin. The nurse should monitor the client for which of the following adverse effects of rifampin?

Red-tinged urine

A nurse is reviewing the lab values of a client who is receiving a continuous IV heparin infusion and has an aPTT of 90 seconds. Which of the following actions should the nurse prepare to take?

Reduce the infusion rate

A nurse is caring for a client who has CA involving the lumbar vertebrae and has been prescribe gabapentin. Which of the following therapeutic effects should the nurse identify for the client when taking this medication?

Reduced cramping, aching, and burning neuropathic painq

A nurse is providing teaching to a client who has cirrhosis and a new prescription for lactulose. The nurse should instruct the client that lactulose has which of the following therapeutic effects?

Reduces ammonia levels

A nurse is caring for a client with DKA who has a prescription for an IV infusion of insulin. The nurse should document that which of the following types of insulin was administered IV to treat ketoacidosis?

Regular insulin

A nurse is preparing to administer IV nitroprusside for a client who had a MI. Which of the following actions should the nurse take?

Regulate the infusion pump rate using the client's weight in the calculation.

A nurse is preparing a discharge teaching plan for a client who is scheduled to begin long term oral prednisone for asthma. Which of the following instructions should the nurse include in the plan?

Schedule the medication on alternate days to decrease adverse effects

A home health nurse is visiting an older adult client who has alzheimer's disease. His caregiver tells the nurse she has been administering prescribed lorazepam 1 mg TID to the client for restlessness and anxiety over the past few days. For which of the following adverse effects should the nurse assess the client?

Sedation

A nurse is developing a teaching plan for a client who has a new prescription for simvastatin. Which of the following instructions should the nurse include in the teaching plan? (Select all that apply.) Report muscle pain to the provider. Avoid taking the medication with grapefruit juice. Take the medication in the early morning. Expect a flushing of the skin as a reaction to the medication. Expect therapy with this medication to be lifelong.

Report muscle pain to the provider Avoid grapefruit juice expect therapy to be lifelong Report muscle pain to the provider is correct. Myopathy is an adverse effect of simvastatin that can lead to rhabdomyolysis. The nurse should instruct the client to report this to the provider. Avoid taking the medication with grapefruit juice is correct. When taken with grapefruit juice, simvastatin increases the risk of muscle injury from elevations in creatine kinase. Take the medication in the early morning is incorrect. This medication is most effective when taken in the evening because cholesterol production generally increases overnight. Expect a flushing of the skin as a reaction to the medication is incorrect. The nurse should identify flushing of the skin as an adverse effect of the medication niacin, which can be used to decrease the client's triglyceride levels. Expect therapy with this medication to be lifelong is correct. If medication therapy is discontinued, cholesterol levels will return to their pretreatment range within several weeks to months.

A nurse is caring for a client who received nalaxone for a suspected opioid overdose. Which of the following findings should the nurse identify as an adverse effect of this medication?

Reports of pain

A nurse is providing teaching to a newly licensed nurse about administering morphine via IV bolus to a client. Which of the following pieces of info should the nurse include in the teaching?

Respiratory depression can occur 7 min after the morphine is administered.

A nurse is caring for a client with asthma who has been taking an inhaled glucocorticoid and long acting beta2 agonist combination DPI for maintenance therapy. The nurse should identify that which of the following is a disadvantage of this medication?

Restricted dosage flexibility

A nurse is caring for a client who has RA and a new prescription for etanercept. Which of the following values should the nurse review prior to the administration of the medication?

Results of last purified protein derivative (PPD) test

A nurse is caring for a client who has asthma and is prescribed a short acting beta2 agonist. Which of the following should the nurse identify as the expected outcome of this medication?

Reverses bronchospasm

A nurse is providing teaching to a group of new parents about medications. The nurse should include that ASA is contraindicated for children who have a viral infection due to the risk of developing which of the following adverse effects?

Reye's syndrome

A nurse is teaching self-administration of NPH insulin to a client who has type 2 DM. Which of the following instructions should the nurse include?

Rotate injection sites within the same area.

A nurse is reviewing laboratory reports for a client who has c diff. and is receiving vancomycin. Which of the following results should the nurse report to the provider before administering the next dose?

Serum creatinine 2.5 mg/dL

A nurse at a clinic is providing follow up care to a client who is taking fluoxetine for depression. Which of the following findings should the nurse identify as an adverse effect of the medication? Tingling toes Sexual dysfunction Absence of dreams Pica

Sexual Dysfunction Sexual dysfunction, including a decreased libido, impotence, and delayed orgasm, or anorgasmia, is a common adverse effect of fluoxetine and occurs in about 70% of clients who take this SSRI antidepressant.

A nurse is caring for a female adult client who is experiencing menopause and has a prescription for estrogen along with progestin. The nurse should identify that the provider has prescribed these medications for which of the following reasons?

Short term use to control urogenital atrophy

A nurse is assessing a client who is receiving a continuous morphine IV infusion and finds the client's respiratory rate has decreased from 20/min to 12/min. Which of the following actions should the nurse take?

Slow the rate of the infusion

A nurse is caring for a client who as cystic fibrosis and has a prescription for high-dose ibuprofen daily. The nurse should identify that which of the following is an expected outcome for the client receiving this medication?

Slowed progression of pulmonary damage

A nurse is caring for a school-aged child who has cystic fibrosis and has been using a corticosteroid inhaler for long-term tx. Which of the following findings should the nurse identify as an adverse effect of long-term use of this medication?

Small stature for age

A nurse is caring for a client who has had a IUD in place for 1 year. Which of the following findings should indicate that the client is experiencing an adverse effect?

Spotting between menses cycles

A nurse is teaching a client who has a new prescription for phenytoin. The nurse should inform the client that which of the following adverse effects can occur with the abrupt withdrawal of phenytoin?

Status epilepticus

A nurse is teaching a client who has chronic stable angina pectoris and a prescription for sublingual nitroglycerin tab. What sequence of instructions should the nurse tell the client to use if he experiences chest pain?

Stop activity, place tab under tongue, wait 5 min, call 911 if pain isn't relieved

A nurse is providing teaching to a client who has a new prescription for sertraline. The client asks the nurse if he should continue to take St. John's wort for depression. Which of the following instructions should the nurse give the client?

Stop taking the herbal supplement while taking this medication

A nurse is teaching a client who has type 2 DM about storing unopened vials of insulin. Which of the following pieces of info should the nurse include in the teaching?

Store the vials in the regrigerator

A nurse is providing discharge teaching about handling medication to a client who is to continue taking oral transmucosal fentanyl raspberry flavored lozenges on a stick. Which of the following information should the nurse include in the teaching? Chew on the medication stick to release the medication. Leave the medication stick in one location of the mouth until melted. Allow the medication 1 hr for analgesia effects to begin. Store unused medication sticks in a storage container.

Store unused medication sticks in storage The nurse should instruct the client to store unused, used, or partially used medication sticks in the safe storage container that comes in the kit when the medication is initially prescribed.

A nurse is providing teaching to a client who has RA and a prescription for long-term prednisone therapy. The nurse should instruct the client to monitor for which of the following adverse effects?

Stress fractures

A nurse manager is instructing a newly licensed nurse about routes of medication administration. Which of the following routes involves medication absorption through the mucous membranes under the tongue?

Sublingual

A nurse is caring for a client who has a prescription for subdermal etonogestrel. The nurse should alert the provider about which of the following findings in the client's medical history?

Takes St. John's wort

A nurse is caring for a female client who has osteoporosis and is taking raloxifene. Which of the following findings should indicate to the nurse that the client is experiencing an adverse effect of this medication?

Sudden onset of dyspnea

A nurse is caring for a client who has a new diagnosis of oral candidiasis after taking tetracycline for 7 days. The nurse should recognize that candidiasis is a manifestation of which of the following adverse effects?

Superinfection

A nurse in a community health clinic is assessing a new client who has prescriptions for isoniazid and rifampin. Whic of the following disorders should the nurse expect the client to have?

TB

A nurse is preparing to administer levothyroxine to a client who has hypothyroidism. The nurse should identify which of the following laboratory results as supporting the administration of this medication?

TSH 8 microunits/mL

A nurse is administering subq epinephrine for a client who is experiencing anaphylaxis. The nurse should monitor the client for which of the following adverse effects?

Tachycardia

A nurse is assessing a client who began taking clozapine 3 weeks ago. Which of the following findings should the nurse report to the provider immediately?

Tachypnea and tachycardia

A nurse is caring for a client who is receiving end-of-life care and has a prescription for fentanyl patches. Which of the following information regarding the adverse effects of fentanyl should the nurse plan to give to the client and family?

Take a stool softener on a daily basis. Constipation is an adverse effect of opioid use. Stool softeners can decrease the severity of this adverse effect.

A nurse in a provider's office is providing teaching to a client with osteoporosis who has a new prescription for alendronate sodium. Which of the following pieces of info should the nurse include?

Take alendronate sodium with a full glass of water on an empty stomach.

A nurse is teaching a client who is taking levothyroxine for hypothyroidism about a new prescription for a calcium supplement. Which of the following pieces of information should the nurse include in the teaching?

Take the calcium supplement 4 hr after taking the levothyroxine

A nurse is reviewing the lab report for a client who is taking tobramycin and notes that the peak blood level is 9.3 mcg/mL. Which of the following actions should the nurse take?

Tell the client that the medication seems to be appropriate

A nurse is teaching a client with type 2 DM about self-administration of a new prescription for acarbose. Which of the following pieces of info should the nurse include in the teaching?

Tell the client to take the medication with food.

A nurse is evaluating a 20 month old child who received a hepatits A immunization 3 days ago. The parent reports that the child has exhibited a loss of appetite following the immunization. Which of the following actions should the nurse take?

Tell the parent that this reaction should only last for a couple of days

A nurse is assessing a client who has schizophrenia and is taking haloperidol. The nurse should report which of the following findings to the provider as a manifestation of neuroleptic malignant syndrome (NMS)? Temperature of 39.7° C (103.5° F) Urinary retention Heart rate 56/min Muscle flaccidity

Temp 103.5 The nurse should report fever to the provider as an indication of NMS, an acute life-threatening emergency. Other manifestations can include respiratory distress, diaphoresis, and either hyper- or hypotension.

A nurse is assessing a client who is receiving clozapine to treat schizophrenia. The nurse should identify an increase in which of the following parameters as an early indication of an adverse effect of this medication?

Temperature

A nurse suspects that a client is having an allergic reaction to a medication. Which of the following factor should the nurse identify as increasing the likelihood of an allergic reaction to the medication?

The client has had previous exposure to the medication

A nurse is assessing a client who has cystic fibrosis. Which of the following pieces of info indicates a therapeutic response to pancreatic enzyme replacement?

The client is having 1-2 BM per day

A nurse is preparing to administer an IM injection for a client. Which of the following factors should the nurse identify as a potential contraindication to administering the medication via the IM route?

The client is taking an anticoagulant

A nurse is planning care for a female client who has a severe IBS-D and a new prescription for alosetron. Which of the following interventions should the nurse include in the plan of care?

The client must sign an agreement with the provider before beginning alosetron.

A nurse is assessing a client 1 hr after administering morphine for pain. The nurse should identify which of the following findings as the best indication that the morphine has been effective? The client's vital signs are within normal limits. The client has not requested additional medication. The client is resting comfortably with eyes closed. The client rates pain as 3 on a scale from 0 to 10.

The client rates pain of 3 on scale 1 to 10 The client's description of the pain is the most accurate assessment of pain.

A nurse administers ceftazidime to a client who has a severe penicillin allergy. The nurse should identify which of the following client findings as an indication that she should complete an incident report?

The client reports shortness of breath A severe penicillin allergy is a contraindication for taking ceftazidime, a cephalosporin antibiotic, due to the potential for cross-sensitivity. Shortness of breath can indicate the client is developing anaphylaxis.

A nurse is caring for a client who is experiencing acute pain and is receiving morphine. Which of the following findings should indicate to the nurse the need to withhold the client's next dose of morphine?

The client's RR is 10/min

A nurse is assessing a client who is postoperative following an outpatient endoscopy procedure using midazolam. The nurse should monitor for which of the following findings as an indication that the client is ready for discharge? The client's capnography has returned to baseline. The client can respond to their name when called. The client is passing flatus. The client is requesting oral intake.

The client's capnography has returned to baseline. The nurse should identify that the client is ready for discharge when the capnography level indicates that gas exchange is adequate.

A nurse in a long-term care facility is administering medications to a group of older adult clients. Which of the following factors of pharmacokinetics should the nurse consider when caring for this age group?

The excretion of medication is reduced.

A nurse is assessing an infant who is scheduled to receive the rotavirus vaccine. Which of the following criteria should the nurse identify as a potential contraindication for administering this vaccine?

The infant has a hx of intussusception

A nurse is reviewing a new prescription for fexofenadine for a 7 yo client who has seasonal allergies. Which of the following findings should the nurse clarify with the provider?

The prescription says to take standard tablets

A nurse is reviewing the medication history of a client who has asthma. Which of the following medication combinations should the nurse identify as incompatible?

Theophylline and zileuton

A nurse is caring for a client who is taking acetazolamide for chronic open angle glaucoma. For which of the following adverse effects should the nurse instruct the client to monitor and report Tingling of fingers Constipation Weight gain Oliguria

Tingling of fingers The nurse should instruct the client to report the adverse effect of paresthesia, a tingling sensation in the extremities, when taking acetazolamide.

A nurse is caring for a client who has HF and is taking oral furosemide 40 mg daily. For which of the following adverse effects should the client be taught to monitor and notify the provider if it occurs?

Tinnitus

A nurse on the acute care unit is caring for a client who is receiving gentamicin IV. The nurse should report which of the following findings to the provider as an adverse effect of the medication? Constipation Tinnitus Hypoglycemia Joint pain

Tinnitus Aminoglycosides, such as gentamicin, are ototoxic, which can manifest as tinnitus and deafness. The nurse should monitor the client for high-pitched ringing in the ears and headaches and should notify the provider if these occur.

A nurse in a clinic is caring for a client who is taking aspirin for the treatment of arthritis. The nurse should identify which of the following findings as an indication that the client is beginning to exhibit salicylism?

Tinnitus Tinnitus is a manifestation of aspirin toxicity, also called salicylism. Other manifestations include sweating, headache, and dizziness.

A nurse is preparing to administer an IV injection to a client. For which of the following reasons should the nurse inject the medication slowly?

To reduce toxicity risk

A nurse is providing teaching to a client who has a new prescription for lisinopril. Which of the following should the nurse include in the teaching as an adverse effect of lisinopril?

Tongue swelling

A nurse is reviewing the medical record of a client who has schizophrenia and prescription for clozapine. Which of the following laboratory tests should the nurse review before administering the medication?

Total cholesterol The nurse should review the client's total cholesterol before administering clozapine, because this medication can cause hyperlipidemia.

A nurse is assessing a client who has hypothyroidism and takes levothyroxine. Which of the following findings indicates that the client is experiencing acute levothyroxine overdose?

Tremors

A nurse is assessing a client who takes oral theophylline for chronic bronchitis relief. The nurse should recognize that which of the following findings indicates toxicity to theophylline?

Tremors

A nurse is caring for a client who is experiencing a seizure while in bed. Which of the following actions should the nurse take?

Turn the client's head to the side

A nurse is reviewing the ECG of a client who is receiving IV furosemide for heart failure. The nurse should identify which of the following findings as an indication of hypokalemia? Tall, tented T-waves Presence of U-waves Widened QRS complex ST elevation

U waves The nurse should identify the presence of U-waves as a manifestation of hypokalemia, an adverse effect of furosemide.

A nurse is caring for a client who is developing acute pulmonary edema and has a new prescription for furosemide 40 mg IV bolus. The nurse should plan to administer the medication using which of the following methods?

Undiluted administered over 2 min

A nurse is assessing a client who reports using several herbal and vitamin supplements daily, including saw palmetto. The nurse should recognize that saw palmeetto is a supplement used by clients to elicit which of the following therapeutic effects?

Urinary health promotion

A nurse is teaching a client who is starting to take amitriptyline. Which of the following findings should the nurse include in the teaching as an adverse effect of the medication?

Urinary retention The nurse should instruct the client that amitriptyline causes the anticholinergic effect of urinary retention.

A nurse in an outpatient facility is assessing a client who is prescribed furosemide 40 mg daily. The client reports taking extra doses to promote weight loss. Which of the following findings should indicate to the nurse that the client is dehydrated?

Urine specific gravity 1.035

A nurse is preparing to administer 100 unites of insulin glargine and 4 units of NPH insulin subcutaneously to a client. Which of the following actions should the nurse plan to take?

Use separate syringes for administering insulin glargine and NPH insulin.

A nurse is assessing a client who has HF and is receiving digoxin. Which of the following findings should indicate to the nurse the client is experiencing digoxin toxicity?

Visual disturbances

A nurse is caring for a client who takes warfarin 2.5 mg PO daily and has an INR of 6.2. The nurse should anticipate a prescription form the provider for which of the following medications?

Vitamin K

A nurse is reviewing the laboratory results of a client who is taking carbamazepine for a seizure disorder. Which of the following findings should the nurse report to the provider?

WBC 3,500/mm3 A WBC count of 3,500/mm3 is below the expected reference range of 5,000 to 10,000/mm3. Leukopenia is an adverse effect of carbamazepine. The nurse should report this finding to the provider and monitor the client for manifestations of infection.

A nurse is preparing to administer medication to a preschooler. The nurse should use which of the following measurements to calculate the medication dosage for this client?

Weight

A nurse is preparing to administer a new prescription of amoxicillin/clavulanic to a client. The client tells the nurse that they are allergic to penicillin. Which of the following actions should the nurse take first?

Withhold the medication. When using the urgent vs nonurgent approach to client care, the nurse should determine that the priority action is to withhold the medication to prevent injury to the client.

A nurse is caring for a client who was recently diagnosed with RA. The nurse should expect the provider to prescribe methotrexate at which of the following times?

Within 3 months of the initial dx

A nurse is providing teaching to a client who is to start therapy with digoxin. For which of the following adverse effects should the nurse instruct the client to monitor and report to the provider?

Yellow - tinged vision The nurse should instruct the client to monitor for and report yellow-tinged vision, which is a sign of digoxin toxicity. Other manifestations of digoxin toxicity include nausea, vomiting, loss of appetite, and fatigue. As the digoxin levels increase, the client can experience cardiac dysrhythmias.

A nurse is caring for a client who has severe asthma and allergic rhinitis. The client is taking theophylline. Which of the following medications should the nurse identify as being incompatible with theophylline?

Zafirlllukast

A nurse administers a dose of metformin to a client instead of the prescribed dose of metoclopramide. Which of the following actions should the nurse take first? Report the incident to the charge nurse. Notify the provider. Check the client's blood glucose. Fill out an incident report.

check blood sugar The first action the nurse should take using the nursing process is to assess the client. The client is at risk for hypoglycemia. The nurse should monitor the client's blood glucose and provide the client with a snack to reduce the risk for hypoglycemia.

A nurse is caring for a client who has hypocalcemia and is receiving calcium citrate. The nurse should identify that which of the following findings indicates a therapeutic response to the medication?

client report of decreased paresthesia. Paresthesia is a manifestation of hypocalcemia. A client report of a decrease in paresthesia is an indication of a therapeutic response to calcium citrate. The nurse should also monitor for a decrease in other manifestations of hypocalcemia, including muscle twitching and cardiac dysrhythmias.

A nurse is caring for a client who has cancer and is taking oral morphine and docusate sodium. The nurse should instruct the client that taking the docusate sodium daily can minimize which of the following adverse effects of morphine? Constipation Drowsiness Facial flushing Itching

constipation Constipation is a common adverse effect of morphine that can be minimized by taking docusate sodium, a stool softener that promotes easier evacuation of stool by increasing water and fat in the intestine.

A nurse is teaching a client who is to start taking hydrocodone with acetaminophen tablets for pain. Which of the following information should the nurse include in the teaching? The medication should be taken 1 hr prior to eating. It takes 48 hr for therapeutic effects to occur. Tablets should not be crushed or chewed. Decreased respirations might occur.

decreased respirations may occur The nurse should instruct the client that hydrocodone with acetaminophen might cause respiratory depression, which is an adverse effect of the medication. The client should avoid taking over-the-counter medications or newly prescribed medications without consulting their provider to avoid increased respiratory depression.

A nurse is monitoring a client who is receiving lactulose for cirrhosis. Which of the following laboratory values related to this medication should indicate to the nurse that the treatment is effective

decreased serum ammonia

A nurse is administering donepezil to a client who has Alzheimer's disease. Which of the following findings should the nurse report to the provider immediately? Dyspepsia Diarrhea Dizziness Dyspnea

dyspnea

A nurse is caring for a client who has sickle cell anemia and is taking hydroxyurea. Which of the following findings should the nurse report to the provider?

hemoglobin 7.0 g/dL A hemoglobin level of 7.0 g/dL indicates hydroxyurea toxicity. This hemoglobin level is below the expected reference range of 14 to 19 g/dL for a male client and 12 to 16 g/dL for a female client. Therefore, the nurse should report this finding to the provider. platelets 75,000/mm3 A platelet level of 75,000/mm3 indicates hydroxyurea toxicity. This platelet level is below the expected reference range of 150,00 to 400,000/mm3. Therefore, the nurse should report this finding to the provider. potassium 5.2 mEq/L A potassium level of 5.2 mEq/L indicates tumor lysis syndrome. This potassium level is above the expected reference range of 3.5 to 5 mEq/L. Therefore, the nurse should report this finding to the provider.

A nurse is assessing a client who is taking propylthiouracil for the treatment of Graves' disease. Which of the following findings should the nurse identify as an indication that the medication has been effective? Hot flashes Urinary retention Constipation Bradycardia

hot flashes The estrogen receptor blocking action of tamoxifen commonly results in the adverse effect of hot flashes.

A nurse is assessing a client who is taking propylthiouracil for the treatment of Graves' disease. Which of the following findings should the nurse identify as an indication that the medication has been effective? Decrease in WBC count Decrease in amount of time sleeping Increase in appetite Increase in ability to focus

increase in ability to focus A client who has Graves' disease can experience psychological manifestations such as difficulty focusing, restlessness, and manic-type behaviors. Propylthiouracil is a thyroid hormone antagonist that decreases the circulating T4 hormone, reducing the manifestations of hyperthyroidism. An increased ability to focus indicates that the medication has been effective.

A nurse is caring for a client who is receiving filgrastim. Which of the following findings should the nurse document to indicate the effectiveness of the therapy? Increased neutrophil count Increased RBC count Decreased prothrombin time Decreased triglycerides

increase neutrophil count Filgrastim stimulates the bone marrow to produce neutrophils. For clients receiving chemotherapy, the risk of infection is minimized.

A nurse is providing teaching to a client who is taking bupropion as an aid to quit smoking. Which of the following findings should the nurse identify as an adverse effect of the medication? Cough Joint pain Alopecia Insomnia

insomnia Bupropion, an atypical antidepressant, has stimulant properties, which can result in agitation, tremors, mania, and insomnia.

A nurse is preparing to administer meperidine to a client who is postop and reports a pain level of 8 on a scale of 0 to 10. Which of the following routes of administration will deliver the medication with the shortest time of onset?

intravenous

A nurse is caring for a client who developed hypoglycemia following an insulin injection. The client is conscious and responds appropriately to verbal stimuli. Which of the following medications should the nurse plan to administer first?

oral glucose tab

A nurse is providing teaching to a client who has a new prescription for ferrous sulfate. The nurse should instruct the client to take the medication with which of the following to promote absorption?

orange juice The absorption of ferrous sulfate is enhanced by a vitamin C source, such as orange juice.

A nurse is caring for a client who is experiencing acute alcohol withdrawal. For which of the following client outcomes should the nurse administer chlordiazepoxide? Minimize diaphoresis Maintain abstinence Lessen craving Prevent delirium tremens

prevent delirium tremens The client should take chlordiazepoxide to prevent delirium tremens during acute alcohol withdrawal.

A nurse is providing teaching to a client who has a gastric ulcer and a new prescription for ranitidine. Which of the following instructions should the nurse include? "Take the medication on an empty stomach for full effectiveness." "You may discontinue this medication when stomach discomfort subsides." "Report yellowing of the skin." "Store the medication in the refrigerator."

report yellowing of the skin Ranitidine can be hepatotoxic and cause jaundice. The nurse should instruct the client to monitor for and report yellowing of the skin or eyes to the provider.

A nurse is providing teaching to a client who is to start treatment for asthma with beclomethasone and albuterol inhalers. Which of the following instructions should the nurse include in the teaching? "Take beclomethasone to avoid an acute attack." "Use beclomethasone 5 minutes before using albuterol." "Limit your calcium and vitamin D intake when taking beclomethasone." "Rinse your mouth after inhaling the beclomethasone."

rinse mouth after The client should rinse their mouth after using beclomethasone, a glucocorticoid inhaler, to prevent oropharyngeal candidiasis and hoarseness. *The client should use the bronchodilator, albuterol, prior to taking beclomethasone, a glucocorticoid inhaler, to enhance its absorption.

A nurse is planning care for a client who is prescribed metoclopramide following bowel surgery. For which of the following adverse effects should the nurse monitor? Muscle weakness Sedation Tinnitus Peripheral edema

sedation Metoclopramide has multiple CNS adverse effects, including dizziness, fatigue, and sedation.

A nurse is providing teaching to a client who has a prescription for ergotamine sublingual to treat migraine headaches. Which of the following information should the nurse include in the instructions? "Take one tablet three times a day before meals." "Take one tablet at onset of migraine." "Take up to eight tablets as needed within a 24-hour period." "Take one tablet every 15 minutes until migraine subsides."

take one tablet on onset of migraine The client should take one tablet immediately after the onset of aura or headache.

A nurse is caring for a client who is in labor. The client is receiving oxytocin by continuous IV infusion with a maintenance IV solution. The external FHR monitor indicates late decelerations. Which of the following actions should the nurse take first? Turn the client to a side-lying position. Disconnect the client's oxytocin from the maintenance IV. Apply oxygen to the client by face mask. Increase the client's maintenance IV infusion rate.

turn to a side lying position The greatest risk to the fetus experiencing late decelerations is injury from uteroplacental insufficiency. Therefore, the priority action the nurse should take is to place the client in a lateral position.


Related study sets

Music Appreciation Unit One Quizzes

View Set

Penny Review : Ovaries and Fallopian Tubes

View Set

Chapter 14--Media Planning and Buying

View Set

FINAL EXAM REVIEW: Nixon, Watergate, Ford, Carter Submit Assignment

View Set

HW3: Homework - Ch.3: Supply and Demand

View Set

Life Insurance Policies - Provisions, Options and Riders 1

View Set